Sie sind auf Seite 1von 148

Name: ___________________________________ Date: ______________

1. First responders, such as firefighters, law enforcement officers, and park rangers, are an integral part of the EMS system because: A) the presence of a person trained to initiate BLS care cannot be ensured. B) they are often trained to assist the advanced-level EMT with procedures. C) the average response time for the EMT crew is approximately 15 minutes. D) they can initiate certain advanced life support procedures before EMS arrival.

2. Which of the following scenarios does NOT involve the administration of advanced life support (ALS)? A) A 48-year-old apneic patient who is intubated by the nasal route B) A 53-year-old cardiac arrest patient who is manually defibrillated C) A 61-year-old trauma patient whose chest is decompressed with a needle D) A 64-year-old cardiac arrest patient who is defibrillated with an AED

3. Which of the following is a unique function of the Emergency Medical Dispatcher (EMD)? A) Relaying relevant information to the EMTs B) Directing the ambulance to the correct address C) Obtaining patient information from the caller D) Providing callers with life-saving instructions

4. What type of medical direction do standing orders and protocols describe? A) Radio B) Online C) Offline D) Direct

5. The continuous quality improvement (CQI) process is designed to: A) identify areas of improvement and provide remedial training if needed. B) ensure that all EMT-Bs maintain certification through the NREMT. C) provide punitive action to EMT-Bs who do not follow local protocols. D) focus specifically on the quality of emergency care provided to the patient.

Page 1

6. EMTs who follow a coordinated continuum of care do so because: A) it maximizes survival and reduces patient suffering. B) tradition demands doing what we have always done. C) the medical community requires progressive phases of care. D) definitive care is most easily provided in a prehospital setting.

7. Obtaining continuing medical education is the responsibility of the: A) State Bureau of EMS. B) EMS training director. C) EMS medical director. D) individual EMT-B.

8. Which of the following is NOT a skill that is provided by the EMT-Basic? A) Initiation of an intravenous line B) Automated external defibrillation C) Administering supplemental oxygen D) Assisting patients with certain medications

9. Which of the following statements MOST accurately describes emergency medical services (EMS)? A) A vast network of advanced life support (ALS) providers who provide definitive emergency care in the prehospital setting B) A team of healthcare professionals who are responsible for providing emergency care and transportation to the sick and injured C) A system composed exclusively of basic-level EMTs who are responsible for providing care to sick and injured patients D) A team of paramedics and emergency physicians who are responsible for providing emergency care to critically injured patients

10. Although transport time to a specialty center may be slightly longer than to a local emergency department: A) specialty center physicians are more highly trained. B) patients must always be taken to the closest hospital. C) the patient will receive definitive care more quickly. D) all patients should be transported to a specialty center.

Page 2

11. You have been working at the scene of a major building collapse for 8 hours. Many injured people are still being removed, and everyone is becoming frustrated and losing focus. This situation is MOST effectively managed by: A) providing large amounts of caffeine to the rescue workers. B) requesting a CISM team to provide on-scene peer support. C) conducting a critical incident stress debriefing the next day. D) allowing each worker to sleep in 15- to 30-minute increments.

12. Determination of exposure is an important part of an exposure control plan because it: A) determines the time of day that most exposures are likely to occur. B) determines which type of communicable diseases might be present in the workplace. C) defines who is most likely to transmit communicable diseases in the workplace. D) defines who is at risk for contact with blood and body fluids and which tasks pose a risk of exposure.

13. You are caring for a 40-year-old female who was involved in a motor-vehicle crash. Her husband, who was driving the vehicle, was killed. When the patient asks you if her husband is all right, you should: A) tell her that he is being resuscitated by other EMT-Bs. B) immediately tell her of his death so that she may grieve. C) let clergy or hospital staff relay the bad news if possible. D) avoid answering her questions and focus on her injuries.

14. Carbon monoxide blocks the ability of the blood to oxygenate the body because it: A) binds with the hemoglobin in the red blood cells. B) causes filling of the lungs with copious secretions. C) causes the body to expel too much carbon dioxide. D) destroys the number of circulating red blood cells.

15. The simplest yet MOST effective method of preventing the spread of an infectious disease is to: A) undergo an annual physical examination. B) ensure that your immunizations are up-to-date. C) wash your hands in between patient contacts. D) undergo HIV and TB testing at least twice a year.

Page 3

16. You are summoned to a convalescent center for an 88-year-old female with an altered mental status. A staff nurse advises you that the patient has terminal cancer and that her physician stated that she would probably die within the next few hours; a valid do not resuscitate (DNR) order is presented to you. When caring for this patient, you should: A) depart the scene and allow her to die with dignity. B) start CPR if she experiences cardiopulmonary arrest. C) provide no interventions and transport to the hospital. D) make her comfortable and provide emotional support.

17. If it is not possible to adequately clean your ambulance at the hospital following a call, you should: A) quickly wipe down all high contact surfaces with an antibacterial solution. B) clean the ambulance at your station in a designated area that is well ventilated. C) wait until the end of your shift and then disinfect the entire patient compartment. D) thoroughly wash the back of the ambulance at a local car wash or similar facility.

18. A critical incident stress debriefing (CISD) should be conducted no longer than ____ hours following the incident. A) 6 B) 12 C) 24 D) 72

19. You are transporting a 40-year-old male with respiratory distress. The patient tells you that he recently had a positive TB skin test and is currently being evaluated for possible tuberculosis. You should: A) apply a nonrebreathing mask on the patient and a HEPA respirator on yourself. B) remain at least 3 feet away from the patient and apply a surgical mask on him. C) apply a sterile surgical mask on yourself and a HEPA respirator on the patient. D) apply a nasal cannula on the patient and a sterile surgical mask on yourself.

20. Critical incident stress management (CISM) can occur at an ongoing scene in all of the following circumstances, EXCEPT: A) when personnel are assessed during periods of rest. B) before personnel are preparing to re-enter the scene. C) when patients are actively being assessed or treated. D) before leaving the scene after the incident is resolved.

Page 4

21. Which of the following statements regarding the Good Samaritan Law is correct? A) It provides the EMT-B immunity from a lawsuit. B) It guarantees that you will not be held liable if sued. C) The law does not protect EMT-Bs who are off duty. D) It will not protect you in cases of gross negligence.

22. Two EMT-Bs witnessed a call in which a coworker gave adequate medical care but ignored the patient's emotional needs. The coworker was deliberately rude solely because the patient was thought to be infected with the human immunodeficiency virus (HIV). The EMT-Bs ignored the coworker's treatment of this patient and took no steps to prevent this behavior from happening again. This lack of action on the part of the two EMT-Bs is considered: A) legal and ethical. B) legal but unethical. C) illegal but ethical. D) illegal and unethical.

23. Which of the following scenarios MOST accurately depicts abandonment? A) Paramedic transfers patient care to an EMT-B B) EMT-I transfers patient care to a paramedic C) Physician assumes patient care from an EMT-B D) EMT-B gives a verbal report to an ER nurse

24. The manner in which the EMT-B must act or behave when caring for a patient is called the: A) EMT-B oath. B) code of ethics. C) standard of care. D) scope of practice.

25. You arrive at the scene of an apparent death. When evaluating the patient, which of the following is a definitive sign of death? A) Absence of a pulse B) Profound cyanosis C) Dependent lividity D) Absent breath sounds

Page 5

26. Maintaining the chain of evidence at the scene of a crime should include: A) quickly moving the weapon out of the patient's sight. B) placing the patient in a private area until the police arrive. C) making brief notes at the scene and then completing them later. D) not cutting through holes in clothing that were caused by weapons.

27. Which of the following types of consent allows treatment of a patient who is unconscious or mentally incapacitated? A) Actual B) Implied C) Informed D) Expressed

28. Which of the following components are needed to prove negligence? A) Abandonment, breach of duty, damages, and causation B) Duty to act, abandonment, breach of duty, and causation C) Duty to act, breach of duty, injury/damages, and causation D) Breach of duty, injury/damages, abandonment, and causation

29. To minimize the risk of litigation, the EMT-B should always: A) transport patients to the hospital of their choice. B) provide competent care that meets current standards. C) use universal precautions with every patient encounter. D) utilize at least four personnel when moving a patient.

30. In which of the following circumstances can the EMT-B legally release confidential patient information? A) A police officer requests a copy to place on file B) The family requests a copy for insurance purposes C) A media representative inquires about the patient D) The patient is competent and signs a release form

31. The firm cartilaginous ring that forms the inferior portion of the larynx is called the: A) costal cartilage. B) cricoid cartilage. C) thyroid cartilage. D) laryngo cartilage.

Page 6

32. Which of the following glands regulates the function of all other endocrine glands in the body? A) Thyroid B) Pituitary C) Adrenal D) Parathyroid

33. As the bronchus divides into smaller air passages, the terminal ends of these smaller passages form the: A) pleura. B) alveoli. C) bronchi. D) capillaries.

34. The distal aspect of the tibia forms the: A) lateral condyle. B) medial malleolus. C) Achilles tendon. D) lateral malleolus.

35. Which of the following anatomic terms is synonymous with the word dorsal? A) Medial B) Posterior C) Palmar D) Anterior

36. What three bones make up the shoulder girdle? A) Clavicle, scapula, humerus B) Acromion, clavicle, scapula C) Acromion, scapula, humerus D) Acromion, humerus, clavicle

37. The most superior portion of the sternum is called the: A) manubrium. B) costal arch. C) angle of Louis. D) xiphoid process.

Page 7

38. The leaf-shaped flap of tissue that prevents food and liquid from entering the trachea during swallowing is called the: A) uvula. B) epiglottis. C) vallecula. D) pharynx.

39. The posterior cartilaginous tip of the sternum is called the: A) sternal notch. B) xiphoid process. C) angle of Louis. D) jugular notch.

40. Which of the following systems is responsible for releasing hormones that regulate body activities? A) Skeletal B) Nervous C) Endocrine D) Reproductive

41. Poor peripheral circulation will cause the skin to appear: A) pink. B) ashen. C) flushed. D) cyanotic.

42. Which of the following scenarios does NOT involve the presence of any symptoms? A) A 44-year-old male with abdominal pain and severe dizziness B) A 49-year-old female with blurred vision and ringing in the ears C) A 55-year-old male with a severe headache and 2 days of nausea D) A 61-year-old female who is unconscious with facial cyanosis

43. Which of the following statements regarding stridor is MOST correct? A) It is a whistling sound heard in the lower airway. B) Stridor is caused by incorrect airway positioning. C) It is a high-pitched, crowing upper airway sound. D) Stridor suggests the presence of fluid in the lungs.

Page 8

44. A 40-year-old male presents with pain to the right upper quadrant of his abdomen. He is conscious and alert with stable vital signs. During your assessment, you note that his skin and sclera are jaundiced. You should suspect: A) acute pancreatitis. B) liver dysfunction. C) gallbladder disease. D) renal insufficiency.

45. Which of the following questions is used to determine a patient's chief complaint? A) "What seems to be the matter?" B) "When did the chest pain begin?" C) "Are you having trouble breathing? D) "Do you have a history of diabetes?

46. A patient with profuse sweating is referred to as being: A) flushed. B) plethoric. C) diaphoretic. D) edematous.

47. When taking a patient's blood pressure in an upper extremity, the diaphragm of the stethoscope is normally placed over which artery? A) Brachial B) Femoral C) Popliteal D) Radial

48. Which of the following is the MOST accurate guide to palpating a pulse? A) Avoid compressing the artery against a bone or solid structure. B) Place the tips of your index and long fingers over the pulse point. C) Use your thumb to increase the surface area that you are palpating. D) Apply firm pressure to the artery with your ring and little fingers.

Page 9

49. When assessing the skin of an unconscious patient, you note that it has a bluish tint to it. This finding is called: A) pallor. B) flushing. C) cyanosis. D) mottling.

50. When you assess capillary refill time (CRT) in an infant, normal color to the tested area should return within: A) 1 second. B) 2 seconds. C) 3 seconds. D) 4 seconds.

51. As you and your partner are carrying a stable patient down a flight of stairs in a stair chair, you feel a sudden, sharp pain in your lower back. You should: A) reposition you hands and continue to move the patient. B) stop the move and request additional lifting assistance. C) guide your partner while moving the chair backwards. D) stop the move and have the patient walk down the stairs.

52. The MOST appropriate carrying device to use when moving a patient across rough or uneven terrain is the: A) stair chair. B) wheeled stretcher. C) scoop stretcher. D) basket stretcher.

53. When the shoulder girdle is aligned over the pelvis during lifting: A) the weight is exerted straight down the vertebrae. B) the hands can be held further apart from the body. C) the muscles of the back experience increased strain. D) the risk of back injuries is significantly increased.

Page 10

54. The extremity lift would NOT be appropriate to use on a patient: A) without a spinal injury. B) with a deformed humerus. C) who complains of nausea. D) with forearm lacerations.

55. Which of the following conditions or situations presents the MOST unique challenge to the EMT-B when immobilizing an elderly patient on a long backboard? A) Joint flexibility B) Patient disorientation C) Naturally deformed bones D) Abnormal spinal curvature

56. Safe lifting of a patient on a cot involves all of the following, EXCEPT: A) using the power-lift if possible. B) bending or flexing at the waist. C) keeping your hands about 10" apart. D) grasping the cot with your palms up.

57. To avoid injury when pushing a patient or other object, you should: A) push the patient from an overhead position if possible. B) avoid pushing the patient with your elbows fully extended. C) push from the area of your body between the knees and hips. D) kneel if you are pushing an object that is above waist level.

58. The direct carry is used to transfer a patient: A) with multiple long bone injuries. B) with a possible cervical spine injury. C) from a bed to the ambulance stretcher. D) who cannot be placed on a backboard.

Page 11

59. A 72-year-old woman lying in her bed complains of crushing chest pain and shortness of breath. She states she has had two previous heart attacks. Her blood pressure is 146/102 mm Hg, pulse is 108 beats/min, and respirations are 28 breaths/min. Which of the following is the MOST appropriate method of transferring this patient to the ambulance? A) Place her on a scoop stretcher and carry her to the ambulance with a minimum of four EMT-Bs. B) Assist her in sitting up on the side of the bed and then use a two-person assist to walk her to the ambulance. C) Immobilize her to a long backboard in a supine position and then transfer the backboard to the wheeled stretcher. D) Use the draw sheet method to transfer her from the bed to the wheeled stretcher and then to the ambulance.

60. When a person is standing upright, the weight of anything being lifted and carried in the hands is FIRST reflected onto the: A) pelvic girdle. B) spinal column. C) thigh muscles. D) shoulder girdle.

61. A 37-year-old male has an apparent foreign body airway obstruction. He is conscious and alert and is coughing forcefully. His skin is pink, warm, and moist. The MOST appropriate treatment for this patient includes: A) a series of back blows and chest thrusts. B) finger sweeps to remove the obstruction. C) performing a series of abdominal thrusts. D) encouraging him to cough and transporting.

62. Which of the following statements regarding normal gas exchange in the lungs is correct? A) The oxygen content in the alveoli is highest during the exhalation phase. B) Oxygen and carbon dioxide diffuse across the alveolar walls and capillaries. C) The actual exchange of oxygen and carbon dioxide occurs in the capillaries. D) Blood that returns to the lungs from the body has a low carbon dioxide content.

63. The jaw-thrust maneuver is used to open the airway of patients with suspected: A) mandibular fractures. B) upper airway swelling. C) cervical spine injuries. D) copious oral secretions.
Page 12

64. The ________ cartilage is a firm ring that forms the inferior part of the larynx. A) cricoid B) thyroid C) laryngeal D) pyriform

65. You are dispatched to a residence where a middle-aged man was found unconscious in his front yard. There are no witnesses that can tell you what happened. You find him in a prone position; his eyes are closed and he is not moving. Your FIRST action should be to: A) palpate for the presence of a carotid pulse. B) log roll him as a unit to a supine position. C) assess the rate and quality of his breathing. D) open his airway with a jaw-thrust maneuver.

66. The exchange of air between the lungs and the environment is called: A) diffusion. B) ventilation. C) respiration. D) oxygenation.

67. The purpose of the pin-indexing system that has been established for compressed gas cylinders is to: A) ensure that the correct regulator is used for the cylinder. B) help you determine what type of oxygen regulator to use. C) prevent destroying or stripping the threads on the cylinder. D) reduce the cylinder pressure to a safe and more useful range.

68. Which of the following would NOT cause a decrease in tidal volume? A) Shallow breathing B) Deep respirations C) Irregular breathing D) Agonal respirations

69. An adult at rest should have a respiratory rate that ranges between: A) 8 and 15 breaths/min. B) 10 and 18 breaths/min. C) 12 and 20 breaths/min. D) 16 and 24 breaths/min.
Page 13

70. As the single EMT-B managing an apneic patient's airway, the preferred initial method of providing ventilations is the: A) mouth-to-mouth technique. B) one-person bag-valve mask device. C) manually triggered ventilation device. D) mouth-to-mask with one-way valve.

71. When performing an ongoing assessment, you should first: A) obtain updated vital signs. B) reassess your interventions. C) repeat the initial assessment. D) confirm medical history findings.

72. An unstable patient should be reassessed every: A) 5 minutes. B) 10 minutes. C) 15 minutes. D) 20 minutes.

73. The "Golden Hour" begins when an injury occurs and ends when: A) the patient receives definitive care. B) the patient is admitted to the ICU. C) you depart the scene for the hospital. D) you arrive at the emergency department.

74. Which of the following signs of respiratory distress is seen MOST commonly in pediatric patients? A) Nasal flaring B) Rapid respirations C) Pursed-lip breathing D) Accessory muscle use

75. When assessing motor function in a conscious patient's lower extremities, you should expect the patient to: A) wiggle his or her toes on command. B) feel you touching the extremity. C) note any changes in temperature. D) identify different types of stimuli.
Page 14

76. When assessing a conscious medical patient, the main purpose of the focused history is to: A) obtain information regarding the patient's present illness. B) determine if the patient's medications caused the illness. C) arrive at a definitive diagnosis of the patient's condition. D) systematically rule out causes of the patient's complaint.

77. Which of the following activities would NOT be performed during the scene size-up? A) Asking a neighbor to secure the patient's dog B) Rapidly assessing a patient's respiratory status C) Notifying the dispatcher to send fire personnel D) Noting the position of a crashed motor vehicle

78. The MOST effective way to gain a patient's trust when he or she is communicating with you is to: A) repeatedly ask questions for clarification. B) ask the same questions to every patient. C) document the patient's words verbatim. D) maintain eye contact and actively listen.

79. If a pulse cannot be palpated at the radial artery, you should: A) attach an AED at once. B) immediately begin CPR. C) assess the carotid pulse. D) elevate the arm 6 to 12.

80. Which of the following statements regarding the focused history and physical exam is correct? A) It will allow you to identify specific problems based on the patient's chief complaint. B) It should be performed en route to the hospital. C) It will help you determine whether an initial assessment should be performed. D) It should only be performed on critical patients.

81. Typical components of an oral patient report include all of the following, EXCEPT: A) the chief complaint or mechanism of injury. B) important medical history not previously given. C) the set of baseline vital signs taken at the scene. D) the patient's response to treatment you provided.

Page 15

82. As you are wheeling your patient through the emergency department doors, you receive another call for a major motor vehicle crash. You should: A) place the patient in a high-visibility area and then respond to the call. B) inform the admissions clerk of the situation and then respond at once. C) leave a copy of the run form with a nurse and then respond to the call. D) respond only after giving a verbal patient report to a nurse or physician.

83. You are transporting a 54-year-old male in cardiac arrest. A first responder is driving the ambulance as you and your partner attempt to resuscitate the patient. What is the MOST logical way of notifying the hospital? A) Call the receiving hospital with your cellular phone as you attempt resuscitation. B) Have the driver contact dispatch and relay the patient information to the hospital. C) Request that a police officer respond to the hospital to apprise them of your arrival. D) Wait until you arrive at the hospital and then quickly apprise them of the situation.

84. When providing a patient report via radio, you should protect the patient's privacy by: A) using coded medical language. B) not disclosing his or her name. C) withholding medical history data. D) refraining from objective statements.

85. Which of the following statements regarding communications with a child is MOST correct? A) Most children are intrigued by strangers wearing uniforms. B) Give the child minimal information to avoid scaring them. C) Standing over a child often increases their level of anxiety. D) Unlike adults, children cannot see through lies or deceptions.

86. Any radio hardware containing a transmitter and receiver that is located in a fixed location is called a: A) repeater. B) multiplex. C) mobile radio. D) base station.

Page 16

87. Medical control gives you an order that seems inappropriate for the patient's condition. After confirming that you heard the physician correctly, you should: A) carry out the order and then carefully document it on the run form. B) advise the physician that the order is unclear and ask for clarification. C) state that you will not carry out the order because it is inappropriate. D) obtain consent from the patient and then carry out the order as usual.

88. Calming and reassuring an anxious patient can be facilitated by: A) maintaining eye contact with the patient whenever possible. B) using medical terminology to ensure the patient understands. C) positioning yourself at a level that is higher than the patient. D) withholding unpleasant information until arrival at the hospital.

89. Two-way communication that requires the EMT-B to "push to talk" and "release to listen" describes what mode of communication? A) Duplex B) Simplex C) Multiplex D) Mediplex

90. Which of the following charges can be brought against you if you injure the reputation of a patient by making derogatory remarks? A) Libel B) Assault C) Slander D) Negligence

91. A 37-year-old male is found unconscious in his car. His airway is patent and his respirations are rapid and labored. As you and your partner are assessing and treating the patient, a police officer hands you a medication named Alupent, which he found in the backseat of the patient's car. This medication suggests that the patient has a history of: A) asthma. B) heart disease. C) hypertension. D) allergic reactions.

Page 17

92. When given to patients with cardiac-related chest pain, nitroglycerin: A) relaxes the walls of the coronary arteries. B) increases myocardial contraction force. C) increases blood return to the right atrium. D) constricts the veins throughout the body.

93. Which of the following medication routes delivers a drug through the skin over an extended period of time, such as a nitroglycerin or nicotine patch? A) Sublingual B) Intraosseous C) Subcutaneous D) Transcutaneous

94. Which of the following is the MOST rapidly acting medication administration route? A) Sublingual B) Intravenous C) Subcutaneous D) Intramuscular

95. Which of the following is an example of a drug's generic name? A) Bayer B) Advil C) Excedrin D) Aspirin

96. Shortly after administering epinephrine to a 29-year-old male with an allergic reaction, the patient complains of a fluttering feeling in his chest. This finding is an example of a/an: A) side effect. B) atypical effect. C) therapeutic effect. D) unpredictable effect.

Page 18

97. A 31-year-old female is experiencing an acute asthma attack. She is conscious and alert, but in obvious respiratory distress. After assisting her with her prescribed metered-dose inhaler, you should: A) check the drug's expiration date to ensure that it is still current. B) contact medical control and apprise him or her of what you did. C) reassess the patient and document her response to the medication. D) administer another treatment in 30 seconds if she is still in distress.

98. A 49-year-old male with an extensive cardiac history presents with 2 hours of crushing chest pain and shortness of breath. He is pale and diaphoretic and tells you that he feels like he's going to die. His medications include nitroglycerin (nitro), Viagra, and Vasotec. His blood pressure is 140/90 mm Hg and his heart rate is 110 beats/min. In addition to administering 100% oxygen, you should: A) obtain physician approval to give the nitroglycerin. B) place him in a supine position and transport at once. C) administer one nitroglycerin and call medical control. D) ask him if he took his Viagra within the past 24 hours.

99. The process of binding or sticking to a surface is called: A) adsorption. B) absorption. C) suspension. D) digestion.

100. Which of the following medication routes has the slowest rate of absorption? A) Rectal B) Ingestion C) Inhalation D) Sublingual

101. Common signs and symptoms of acute hyperventilation syndrome include: A) altered mental status and bradycardia. B) unilateral paralysis and slurred speech. C) anxiety, dizziness, and severe bradypnea. D) tachypnea and tingling in the extremities.

Page 19

102. You receive a call for a 70-year-old female with respiratory distress. Her husband tells you that she has congestive heart failure; however, he does not think that she has been taking her medications as prescribed. The patient is laboring to breathe, appears tired, and has cyanosis around her lips. You should: A) begin assisting her ventilations with a BVM. B) apply a pulse oximeter and obtain vital signs. C) administer oxygen via a nonrebreathing mask. D) obtain a complete list of all of her medications.

103. When the level of arterial carbon dioxide increases: A) the brain stem inhibits respirations. B) respirations increase in rate and depth. C) exhalation lasts longer than inhalation. D) respirations decrease in rate and depth.

104. Which of the following is MOST characteristic of adequate breathing? A) 22 breaths/min with an irregular pattern of breathing and cyanosis B) 20 breaths/min with shallow movement of the chest wall and pallor C) 24 breaths/min with bilaterally equal breath sounds and pink skin D) 30 breaths/min with supraclavicular retractions and clammy skin

105. Harsh, high-pitched inspiratory sounds are MOST characteristic of: A) rales. B) stridor. C) rhonchi. D) wheezing.

106. A pleural effusion is MOST accurately defined as: A) a unilaterally collapsed lung. B) diffuse collapsing of the alveoli. C) fluid accumulation outside the lung. D) a bacterial infection of the lung tissue.

107. Dyspnea is MOST accurately defined as: A) shortness of breath or difficulty breathing. B) a complete cessation of respiratory effort. C) a marked increase in the exhalation phase. D) labored breathing with reduced tidal volume.
Page 20

108. A conscious and alert 29-year-old female with a history of asthma complains of difficulty breathing that began after her morning jog. The temperature outside is 40F. On exam, you hear bilateral expiratory wheezing. After providing 100% oxygen, you should: A) place her in a recumbent position to facilitate breathing. B) contact medical control and administer an antihistamine. C) call medical control and ask how to proceed with treatment. D) determine if she has been prescribed a beta-agonist inhaler.

109. A 59-year-old male with a history of emphysema complains of an acute worsening of his dyspnea and pleuritic chest pain following a forceful cough. Your assessment reveals that he has a barrel-shaped chest, unilaterally diminished breath sounds, and tachycardia. What is the MOST likely cause of this patient's condition? A) Rupture of the diaphragm B) Exacerbation of his COPD C) Acute pulmonary embolism D) Spontaneous pneumothorax

110. In what area of the lungs does respiration occur? A) Alveoli B) Trachea C) Bronchi D) Capillaries

111. The principle clinical difference between a stroke and hypoglycemia is that patients with hypoglycemia: A) always take oral medications to maintain normal blood glucose levels. B) do not present with slurred speech or weakness to one side of the body. C) are typically alert and attempt to communicate with healthcare providers. D) usually have an altered mental status or decreased level of consciousness.

112. A patient without a history of seizures experiences a sudden convulsion. The LEAST likely cause of this seizure is: A) epilepsy. B) brain tumor. C) serious infection. D) intracranial bleeding.

Page 21

113. The most basic functions of the body, such as breathing, blood pressure, and swallowing, are controlled by the: A) brain stem. B) cerebrum. C) cerebellum. D) cerebral cortex.

114. Interruption of cerebral blood flow may result from all of the following, EXCEPT: A) a thrombus. B) an embolism. C) cerebral vasodilation. D) an acute arterial rupture.

115. Muscle control and body coordination are controlled by the: A) cerebrum. B) cerebellum. C) brain stem. D) cerebral cortex.

116. Which of the following conditions would be the LEAST likely to mimic the signs and symptoms of a stroke? A) Hypoglycemia B) Hypovolemia C) Postictal state D) Intracranial bleeding

117. Which of the following medications is NOT used to treat patients with a history of seizures? A) Dilantin B) Dilaudid C) Tegretol D) Phenobarbital

Page 22

118. A 58-year-old male presents with confusion, right-sided weakness, and slurred speech. As your partner is applying oxygen, it is MOST important for you to: A) list all of the patient's current medications. B) obtain a complete set of baseline vital signs. C) administer glucose to rule out hypoglycemia. D) ask his wife when she noticed the symptoms.

119. You respond to a residence for a child who is having a seizure. Upon arrival at the scene, you enter the residence and find the mother holding her child, a 2-year-old male. The child is conscious and crying. According to the mother, the child had been running a high fever and then experienced a seizure that lasted approximately 3 minutes. You should: A) cover the child with wet towels and give oxygen via nasal cannula. B) transport the child to the hospital and reassure the mother en route. C) advise the mother to take her child to the doctor the following day. D) call medical control and request permission to give the child Tylenol.

120. When obtaining medical history information from the family of a suspected stroke patient, it is MOST important to determine: A) when the patient last appeared normal. B) if there is a family history of a stroke. C) if the patient has been hospitalized before. D) the patient's overall medication compliance.

121. A 66-year-old female complains of diffuse abdominal pain and nausea. She has a history of hypertension and occasional depression. When caring for this patient, you should be MOST concerned with: A) the possibility that she may vomit. B) determining the origin of her pain. C) the emotional effects caused by the pain. D) an acute rise in her systolic blood pressure.

122. The parietal peritoneum lines the: A) retroperitoneal space. B) lungs and chest cavity. C) walls of the abdominal cavity. D) surface of the abdominal organs.

Page 23

123. A young female presents with costovertebral angle tenderness. She is conscious and alert with stable vital signs. Which of the following organs is MOST likely causing her pain? A) Liver B) Kidney C) Pancreas D) Gallbladder

124. A 35-year-old mildly obese woman is complaining of localized pain in the right upper quadrant with referred pain to the right shoulder. The MOST likely cause of her pain is: A) acute cystitis. B) cholecystitis. C) appendicitis. D) pancreatitis.

125. A 59-year-old male presents with a sudden onset of severe lower back pain. He is conscious and alert, but very restless and diaphoretic. Your assessment reveals a pulsating mass to the left of his umbilicus. You should: A) vigorously palpate the abdomen to establish pain severity. B) administer oxygen and prepare for immediate transport. C) place the patient in a sitting position and transport at once. D) request a paramedic unit to give the patient pain medication.

126. Pain that is localized to the lower back and/or lower abdominal quadrants is MOST suggestive of: A) acute pancreatitis. B) an aortic aneurysm. C) a kidney infection. D) acute appendicitis.

127. Pain felt at a location other than its origin is called: A) referred pain. B) radiating pain. C) visceral pain. D) remote pain.

Page 24

128. Which of the following conditions is more common in women than in men? A) Cystitis B) Hepatitis C) Pancreatitis D) Cholecystitis

129. You are caring for a 25-year-old female with acute left lower quadrant pain. She is conscious and alert and in obvious pain. Her blood pressure is 110/70 mm Hg, pulse is 100 beats/min and regular, and respirations are 24 and unlabored. In addition to administering oxygen, you should: A) place her in Trendelenburg's position. B) request permission to give her Motrin. C) give her small sips of water to drink. D) place her in a comfortable position.

130. Which of the following is a nonspecific complaint that is commonly associated with an acute abdomen? A) High fever B) Hematemesis C) Loss of appetite D) Lower quadrant pain

131. A 49-year-old male presents with confusion, sweating, and visual hallucinations. The patient's wife tells you that he is a heavy drinker and she thinks he had a seizure shortly before your arrival. This patient is MOST likely experiencing: A) acute hypovolemia. B) alcohol intoxication. C) acute schizophrenia. D) delirium tremens (DTs).

132. You respond to a college campus for a young male who is acting strange. After law enforcement has secured the scene, you enter the patient's dorm room and find him sitting on the edge of the bed; he appears agitated. As you approach him, you note that he has dried blood around both nostrils. He is breathing adequately, has a rapid, irregular pulse, and his BP is 200/110 mm Hg. Treatment for this patient includes: A) requesting a paramedic to administer naloxone (Narcan). B) assisting his ventilations with a BVM and 100% oxygen. C) asking law enforcement to place handcuffs on the patient. D) attempting to calm him and giving him oxygen if tolerated.
Page 25

133. Syrup of ipecac is no longer recommended to treat patients who have ingested a poisonous substance because it: A) has been linked to hypotension. B) does not effectively induce vomiting. C) may result in aspiration of vomitus. D) has toxic effects on the myocardium.

134. An overdose on acetaminophen (Tylenol) will MOST likely cause: A) liver failure. B) gastric ulcers. C) kidney failure. D) CNS depression.

135. The usual dose for activated charcoal is up to ___ g for a pediatric patient and up to ___ g for an adult patient. A) 5, 10 B) 10, 20 C) 12.5, 25 D) 25, 50

136. After administering activated charcoal to a patient, it is MOST important to: A) call medical control. B) be alert for vomiting. C) reassess the patient's BP. D) document the intervention.

137. You respond to a local motel for a young female who was sexually assaulted. The patient is conscious but confused. She tells you that the last thing she remembers was drinking beer at a club with her friends the night before. When she awoke, she was in the bed of the motel room. You should be MOST suspicious that this patient: A) is a heroin abuser. B) is acutely intoxicated. C) was given Rohypnol. D) is abusing marijuana.

Page 26

138. The poison control center will be able to provide you with the MOST information regarding the appropriate treatment for a patient with a drug overdose if they: A) know the location of the closest hospital. B) are aware of the patient's age and gender. C) are aware of the substance that is involved. D) know why the patient overdosed on the drug.

139. A 3-year-old female ingested several leaves from a plant in the living room. The child's mother is not sure what type of plant it is, stating that she bought it simply because it was pretty. After completing your initial assessment of the child, you should: A) administer 25 g of activated charcoal. B) induce vomiting with syrup of ipecac. C) contact the regional poison control center. D) immediately transport the child to the hospital.

140. Activated charcoal is given to patients who have ingested certain substances because it: A) induces vomiting and empties the stomach. B) binds to the substance and prevents absorption. C) decreases absorption of poisons into the lungs. D) is a direct antidote for many toxic substances.

141. When assessing a hypothermic patient, you should palpate for a carotid pulse for approximately __________ seconds before determining that he or she is pulseless. A) 10 to 20 B) 20 to 30 C) 30 to 45 D) 60 to 90

142. A person's ability to shiver is lost when his or her body temperature falls below: A) 90F. B) 92F. C) 95F. D) 97F.

Page 27

143. Hyperthermia is MOST accurately defined as a condition in which: A) the core body temperature exceeds 98.6F. B) the body is exposed to more heat than it can lose. C) heat evaporates a significant amount of body water. D) the body eliminates more heat than it can generate.

144. To obtain the MOST accurate reading of a patient's core body temperature, you should place a special hypothermia thermometer: A) into the patient's rectum. B) under the patient's tongue. C) behind the patient's knee. D) under the patient's armpit.

145. Which of the following conditions would be the LEAST likely to increase a person's risk of hypothermia? A) Hypoperfusion B) Severe infection C) Hyperglycemia D) Spinal cord injury

146. Hypothermia can worsen internal bleeding secondary to: A) cardiac arrhythmias. B) a decreased heart rate. C) severe muscular rigidity. D) blood-clotting abnormalities.

147. A 48-year-old male was stung on the leg by a jellyfish while swimming in the ocean. He is conscious and alert, but complains of intense pain at the wound site. Specific treatment for this patient includes: A) irrigating the wound with vinegar and immersing his leg in hot water. B) pulling the nematocysts out with tweezers and bandaging the wound. C) immersing his leg in fresh cold water and scraping away the stingers. D) applying a chemical ice pack to the wound and encouraging movement.

Page 28

148. Which of the following statements regarding drowning is MOST correct? A) Hypoxia in the drowning victim initially occurs due to water in the lungs. B) Artificial ventilations can easily be performed in patients with a laryngospasm. C) Large amounts of water enter the lungs in a small number of drowning victims. D) Laryngospasm following submersion in water makes rescue breathing difficult.

149. Which of the following statements regarding lightning strikes is MOST correct? A) Lighting often results in a brief period of asystole that resolves spontaneously. B) The tissue damage pathway caused by lightning usually occurs through the skin. C) Victims who are struck by lightning often experience severe full-thickness burns. D) Cervical spine fractures are the most common cause of lightning-related deaths.

150. Compared to adults, infants and children are at higher risk for hypothermia for all of the following reasons, EXCEPT: A) a decreased ability to shiver. B) a relatively small surface area. C) a smaller amount of body fat. D) a smaller overall muscle mass.

151. From a mental health standpoint, an abnormal or disturbing pattern of behavior is a matter of concern if it lasts for at least: A) 1 week. B) 2 weeks. C) 3 weeks. D) 4 weeks.

152. Which of the following conditions or factors would be the LEAST likely to result in a change in behavior? A) Low blood glucose levels B) Antihypertensive medications C) Exposure to excess heat or cold D) Inadequate blood flow to the brain

Page 29

153. Which of the following terms is defined as a manner in which a person responds to his or her environment? A) Affect B) Behavior C) Neurosis D) Psychosis

154. Which of the following statements regarding the detailed physical exam of a patient with a behavioral emergency is MOST correct? A) It is critical that the EMT-B performs a detailed physical exam on all patients with a behavioral emergency in order to detect potential underlying problems. B) The detailed physical exam may be detrimental to gaining the patient's trust and should not be performed unless there is an accompanying physical complaint. C) The detailed physical exam is an integral part of the assessment process of a patient with a behavioral emergency, but it should be performed en route to the hospital. D) Performing a detailed physical exam on a patient with a behavioral emergency will likely cause him or her to become agitated or violent and should not be performed.

155. The term behavioral crisis is MOST accurately defined as: A) a sudden, violent outburst of an otherwise mentally stable person toward a family member. B) any reaction that interferes with activities of daily living or is deemed unacceptable by others. C) a situation in which a patient demonstrates bizarre behavior and becomes a risk to other people. D) a period of severe depression that lasts longer than 2 weeks and cannot be controlled with medications.

156. Organic brain syndrome (OBS) is MOST accurately defined as: A) bizarre behavior secondary to a chemical imbalance or disturbance in the brain. B) a dysfunction of the brain caused by abnormal physical or physiological function. C) a disorder that cannot be traced to the abnormal structure or function of an organ. D) a change in behavior or mental status secondary to inadequate cerebral blood flow.

Page 30

157. You are assessing a 45-year-old female who is severely depressed. She states that it seems as though her entire world is crashing down around her. She further states that she has had frequent thoughts of suicide, but is not sure if she can actually go through with it. How should you manage this situation? A) Ask the patient if she has developed a suicidal plan. B) Leave the scene and have a neighbor check in on her. C) Have law enforcement place her in protective custody. D) Encourage the patient to remain quiet during transport.

158. In contrast to a behavioral crisis, a psychiatric emergency occurs when a person: A) demonstrates agitation or violence or becomes a threat to self or others. B) experiences feelings of sadness and despair for greater than one month. C) exhibits impaired functioning due to a chemical or genetic disturbance. D) experiences a sudden attack of panic secondary to a stressful situation.

159. In addition to ensuring his or her own safety, the EMT-B's responsibility when caring for a patient with a behavioral emergency is to: A) defuse and control the situation and safely transport the patient. B) diagnose the patient's problem and provide definitive treatment. C) transport the patient directly to a specialized psychiatric facility. D) determine the underlying cause of the problem and offer advice.

160. A 78-year-old female presents with an acute change in her behavior. The patient's son tells you that his mother has a history of type II diabetes and was diagnosed with Alzheimer's disease 6 months ago. The patient's speech is slurred and she is not alert to her surroundings. You should: A) transport the patient to a psychiatric facility. B) inquire about the possibility of head trauma. C) conclude that the patient's blood sugar is high. D) allow the patient to refuse transport if she wishes.

161. Bones are connected to other bones by tough fibrous tissues called: A) bursa. B) tendons. C) cartilage. D) ligaments.

Page 31

162. During your assessment of a 29-year-old female with significant deformity to her left elbow, you are unable to palpate a radial pulse. Your transport time to the hospital is approximately 40 minutes. You should: A) splint the elbow in the position of deformity and transport immediately. B) apply gentle manual traction in line with the limb and reassess for a pulse. C) carefully straighten the injured arm and secure it with padded board splints. D) make two or three attempts to restore distal circulation by manipulating the elbow.

163. Assessing a person's neurovascular status following a musculoskeletal injury includes all of the following, EXCEPT: A) assessing motor function. B) assessing sensory function. C) evaluating proximal pulses. D) determining capillary refill.

164. Which of the following fractures has the greatest potential for internal blood loss and shock? A) Hip B) Femur C) Pelvis D) Humerus

165. When assessing a patient with a possible fracture of the leg, the EMT-B should: A) assess proximal circulation. B) compare it to the uninjured leg. C) carefully move it to elicit crepitus. D) ask the patient to move the injured leg.

166. The act of pulling on a body structure in the direction of its normal alignment is called: A) traction. B) reduction. C) stabilization. D) immobilization.

Page 32

167. Skeletal muscle is attached to the bone by tough, ropelike fibrous structures called: A) fascia. B) tendons. C) cartilage. D) ligaments.

168. Traction splints are used primarily to immobilize and secure fractures of the: A) hip. B) pelvis. C) femur. D) humerus.

169. Prior to splinting an open extremity fracture, the EMT-B should first: A) assess the injury for crepitation. B) manually stabilize the injury site. C) assess distal neurovascular status. D) cover all bleeding or open wounds.

170. A hip fracture is actually a fracture of the: A) pelvic girdle. B) femoral shaft. C) pubic symphysis. D) proximal femur.

171. A 45-year-old male was working on his roof when he fell approximately 12 feet, landing on his feet. He is conscious and alert and complains of an ache in his lower back. He is breathing adequately and has stable vital signs. You should: A) obtain a Glasgow Coma Score value and give him oxygen. B) perform a focused physical exam and immobilize his spine. C) allow him to refuse transport if his vital signs remain stable. D) perform a rapid trauma assessment and immobilize his spine.

172. If you do not have the appropriate size cervical collar, you should: A) use rolled towels to immobilize the patient's head. B) place sandbags on either side of the patient's head. C) ask the patient to keep his or her head in a neutral position. D) defer cervical immobilization and apply lateral head blocks.

Page 33

173. Which of the following statements regarding cervical collars is MOST correct? A) Once a cervical collar is applied, you can cease manual head stabilization. B) A cervical collar is used in addition to, not instead of, manual immobilization. C) Cervical collars are contraindicated in patients with numbness to the extremities. D) The patient's head should be forced into a neutral position to apply a cervical collar.

174. A football player experienced a possible spinal injury when he was tackled. He is conscious and alert, but tells you that he is having trouble breathing. His respirations are 28 breaths/min and labored. He is still wearing his helmet. You should: A) leave the helmet on, secure him to a long backboard, and give oxygen. B) ask him to carefully remove the helmet as you support his head and neck. C) leave the helmet on but be prepared to remove it quickly if he deteriorates. D) carefully remove the helmet, immobilize his spine, and administer oxygen.

175. When opening the airway of a patient with a suspected spinal injury, you should use the: A) tongue-jaw lift. B) head-tilt neck-lift. C) head-tilt chin-lift. D) jaw-thrust maneuver.

176. When immobilizing a patient on a long backboard, you should: A) have the patient exhale before fastening the torso straps. B) secure the torso and then center the patient on the board. C) follow the commands of the person at the patient's torso. D) ensure that you secure the torso before securing the head.

177. The effectiveness of positive-pressure ventilations when treating a head-injured patient can ONLY be determined by: A) immediate reassessment following the intervention. B) a neurosurgeon or emergency department physician. C) reassessing the patient's BP after at least 10 minutes. D) noting a decrease in the heart rate during ventilations.

178. The MOST common and serious complication of a significant head injury is: A) skull fracture. B) cerebral edema. C) hypoxic seizures. D) acute hypotension.
Page 34

179. Coordination of body movement is controlled by the: A) medulla. B) cerebrum. C) cerebellum. D) brain stem.

180. Common signs of a skull fracture include all of the following, EXCEPT: A) mastoid process bruising. B) ecchymosis around the eyes. C) noted deformity to the skull. D) superficial scalp lacerations.

Midyear exam practice 09. This includes chapters 1,2,3,4,5,6,7,8,9,10,11,13,14,18,19,29,30 but NOT 20 and NOT CPR.

Page 35

Answer Key - midyear exam practice 09 1. 2. 3. 4. 5. 6. 7. 8. 9. 10. 11. 12. 13. 14. 15. 16. 17. 18. 19. 20. 21. 22. 23. 24. 25. 26. 27. 28. 29. 30. 31. 32. 33. 34. 35. 36. 37. 38. 39. 40. 41. A D D C A A D A B C B D C A C D B D A C D B A C C D B C B D B B B B B A A B B C B
Page 36

42. 43. 44. 45. 46. 47. 48. 49. 50. 51. 52. 53. 54. 55. 56. 57. 58. 59. 60. 61. 62. 63. 64. 65. 66. 67. 68. 69. 70. 71. 72. 73. 74. 75. 76. 77. 78. 79. 80. 81. 82. 83. 84.

D C B A C A B C B B D A B D B B C D D D B C A B B A B C D C A A A A A B D C A C D B B
Page 37

85. 86. 87. 88. 89. 90. 91. 92. 93. 94. 95. 96. 97. 98. 99. 100. 101. 102. 103. 104. 105. 106. 107. 108. 109. 110. 111. 112. 113. 114. 115. 116. 117. 118. 119. 120. 121. 122. 123. 124. 125. 126. 127.

C D B A B C A A D B D A C D A B D A B C B C A D D A D A A C B B B D B A A C B B B B A
Page 38

128. 129. 130. 131. 132. 133. 134. 135. 136. 137. 138. 139. 140. 141. 142. 143. 144. 145. 146. 147. 148. 149. 150. 151. 152. 153. 154. 155. 156. 157. 158. 159. 160. 161. 162. 163. 164. 165. 166. 167. 168. 169. 170.

A D C D D C A D B C C C B C A B A C D A D A B D B B B B B A A A B D B C C B A B C D D
Page 39

171. 172. 173. 174. 175. 176. 177. 178. 179. 180.

B A B D D D A B C D

Page 40

Name: ___________________________________ FHS EMT midyear exam 2011. Please place all final answers on the answer sheet and turn it in with the test booklet. There are 3 bonus items at the end; you may only complete two (2) for points.
1. As an EMT-B, you may be authorized to administer aspirin to a patient with chest pain

based on: A) the patient's condition. B) your local EMS protocols. C) the national standard of care. D) transport time to the hospital.
2. EMTs who follow a coordinated continuum of care do so because: A) it maximizes survival and reduces patient suffering. B) tradition demands doing what we have always done. C) the medical community requires progressive phases of care. D) definitive care is most easily provided in a prehospital setting. 3. The chemical and physical reactions commonly referred to as the fight or flight response

involve interaction of the: A) nervous and endocrine systems. B) respiratory and nervous systems. C) gastrointestinal and nervous systems. D) cardiovascular and endocrine systems.
4. What type of stress reaction occurs when an EMT-B is exposed to many insignificant

stressors over a period of several months or years? A) Acute stress reaction B) Cumulative stress reaction C) Posttraumatic stress reaction D) Critical incident stress reaction
5. What should you do before attempting to access a patient trapped in a vehicle? A) Check for other patients. B) Ensure the vehicle is stable. C) Request another ambulance. D) Contact medical control.

Page 1

6. Which of the following statements about communicable diseases is correct? A) Not all people who are infected with a communicable disease show signs or symptoms

of illness. B) Anyone who has been exposed to a communicable disease always is a carrier of that disease. C) Anyone who is infected with a communicable disease shows signs or symptoms of that disease. D) Any time you take the temperature of a patient who has a communicable disease, you need to wear gloves or you will become infected.
7. While on duty, your partner asks you out on a date and touches you in an inappropriate

location without your consent. You should: A) tell your partner to quit kidding around and focus on his or her job. B) warn your partner that you will report him or her if it happens again. C) notify law enforcement personnel and have your partner arrested. D) tell your partner to stop and report the incident to your supervisor.
8. Which of the following scenarios MOST accurately depicts informed consent? A) A patient advises an EMT-B of why he or she is refusing care B) An EMT-B advises a patient of the risks of receiving treatment C) An EMT-B initiates immediate care for an unconscious adult D) A patient is advised by an EMT-B of the risks of refusing care 9. The normal stimulus to breathe in a healthy patient is based on: A) low levels of oxygen. B) high levels of oxygen. C) low levels of carbon dioxide. D) high levels of carbon dioxide. 10. Contraction of the right ventricle causes: A) closure of the mitral and aortic valves. B) a return of blood from the pulmonary veins. C) blood to flow into the pulmonary circulation. D) ejection of blood into the systemic circulation. 11. Which of the following is NOT a function of the skin? A) Sensory reception B) Temperature regulation C) Metabolic coordination D) Pressure and pain perception
Page 2

12. The central nervous system is composed of the: A) brain and spinal cord. B) brain and sensory nerves. C) motor and sensory nerves. D) spinal cord and sensory nerves. 13. Deoxygenated blood from the abdomen, pelvis, and lower extremities is returned to the

right atrium via the: A) common iliac vein. B) coronary sinus vein. C) inferior vena cava. D) superior vena cava.
14. The normal respiratory rate for an adult should range from: A) 10 to 15 breaths per minute. B) 12 to 20 breaths per minute. C) 18 to 24 breaths per minute. D) 24 to 28 breaths per minute. 15. A palpable pulse is created by: A) the pressure of circulating blood against the walls of the arteries. B) the pressure that is caused when venous blood returns to the heart. C) pressure waves through the arteries caused by cardiac contraction. D) electrical conduction in the heart producing ventricular contraction. 16. When assessing the skin of a patient, you note that it has a bluish tint to it. This finding is

called: A) pallor. B) flushing. C) cyanosis. D) mottling.


17. Bradycardia is defined as a pulse rate less than _____ beats per minute and tachycardia is

defined as a heart rate greater than _____ beats per minute. A) 60, 100 B) 50, 110 C) 40, 120 D) 30, 130

Page 3

18. The topographic term used to describe the location of body parts that are closer toward

the midline or center of the body is: A) lateral. B) medial. C) midaxillary. D) midclavicular.
19. What section of the Health Insurance Portability and Accountability Act (HIPAA) MOST

affects EMS personnel? A) Controlling insurance cost B) Protecting patient privacy C) Preventing insurance fraud D) Ensuring access to insurance
20. When caring for a 65-year-old male with respiratory distress, you place him in a

comfortable position, but do not apply oxygen. The patient's condition continues to deteriorate and he develops cardiac arrest and dies at the hospital. This scenario is an example of: A) assault. B) battery. C) negligence. D) abandonment.
21. You respond to the home of a 59-year-old man who is unconscious, has slow, shallow

breathing, and a weak pulse. The family states that the patient has terminal brain cancer and does not wish to be resuscitated. They further state that there is a do not resuscitate (DNR) order for this patient; however, they are unable to locate it. You should: A) begin treatment and contact medical control as needed. B) honor the patient's wishes and withhold all treatment. C) transport the patient without providing any treatment. D) decide on further action once the DNR order is produced.
22. You suspect that a pregnant 16-year-old girl has a broken leg after she was hit by a car.

You explain that you plan to splint her leg, and she agrees to treatment. What type of consent is her agreement considered? A) Implied B) Informed C) Expressed D) Minor's

Page 4

23. Which of the following types of consent allows treatment of a patient who is unconscious

or mentally incapacitated? A) Actual B) Implied C) Informed D) Expressed


24. In relation to the wrist, the elbow is: A) distal. B) medial. C) lateral. D) proximal. 25. Twelve pairs of ribs attach to what section of the spinal column? A) Sacral B) Lumbar C) Thoracic D) Coccyx 26. The cervical spine is composed of ____ vertebrae. A) 4 B) 5 C) 6 D) 7 27. Which of the following organs or structures lies within the retroperitoneal space? A) Spleen B) Kidneys C) Gallbladder D) Abdominal aorta 28. You can auscultate a patient's blood pressure by: A) comparing the pulse of the wrist with the pulse in the neck. B) feeling for the return of a pulse while inflating the blood pressure cuff. C) deflating a blood pressure cuff while watching the needle of the gauge bounce. D) deflating the cuff while listening with a stethoscope to the return of blood flow through

the brachial artery.

Page 5

29. During your assessment of a 6-month-old male with vomiting and diarrhea, you note that

his capillary refill time is approximately 4 seconds. From this information, you can conclude that his: A) respiratory status is adequate. B) systolic blood pressure is normal. C) peripheral perfusion is decreased. D) skin temperature is abnormally cold.
30. You arrive on scene for a 37 year-old male patient with difficulty breathing. "The scene is

safe; BSI." You enter the residence and find the patient sitting in a chair in obvious respiratory distress. You first actions in order are: A) ask the patient what's wrong, count respirations, and apply oxygen. B) obtain a set of baseline vital signs, obtain medical history, and transport. C) assess the patient's ABCs and vital signs and document them. D) introduce yourself to the patient, ask what's wrong, and apply oxygen.
31. In order to facilitate a safe and coordinated move, the team leader should: A) be positioned at the feet so the team can hear. B) use preparatory commands to initiate any moves. C) speak softly but clearly to avoid startling the patient. D) never become involved in the move, just direct the move. 32. To minimize the risk of injuring yourself when lifting or moving a patient, you should: A) flex at the waist instead of the hips. B) avoid the use of log rolls or body drags. C) use a direct carry whenever it is possible and bend your elbows. D) keep your spine erect and primarily use your leg strength. 33. You arrive at the scene of a motor-vehicle crash, only one patient still seated in his car.

There are no scene hazards. BSI. You approach the vehicle and note the patient is semiconscious with a large laceration to his forehead. You should FIRST: A) direct your partner to apply manual in-line support of the patient's head. B) apply a cervical collar and quickly remove the patient with a clothes drag. C) apply a KED before attempting to move the patient. D) slide a long backboard under his buttocks and secure him supine to the board.

Page 6

34. A 19-year-old female is found unconscious by her roommate. Your initial assessment

reveals that her breathing is shallow and slow. You insert an oral airway and she does not gag. You should: A) remove the airway and suction her oropharynx. B) leave the airway in place, apply BVM, and be prepared to suction her oropharynx. C) insert the airway no further but prepare to suction her oropharynx. D) immediately insert a nasopharygeal airway and apply BVM.
35. Which of the following activities would NOT be performed during the scene size-up? A) Asking a neighbor to secure the patient's dog B) Rapidly assessing a patient's respiratory status C) Notifying the dispatcher to send fire personnel D) Noting the position of a crashed motor vehicle 36. When a patient's respirations are shallow: A) chest rise will be easily noticeable. B) tidal volume is markedly reduced. C) oxygenation occurs more efficiently, D) carbon dioxide elimination is increased. 37. The "Golden Hour" begins when an injury occurs and ends when: A) the patient receives definitive care. B) the patient is admitted to the ICU. C) you depart the scene for the hospital. D) you arrive at the emergency department. 38. The presence of subcutaneous emphysema could indicate all of the following conditions,

EXCEPT: A) pneumothorax. B) crushed trachea. C) laryngeal fracture. D) myocardial rupture.


39. You are transporting a 49-year-old male with chest pain and an extensive cardiac history

when he suddenly loses consciousness. You should: A) quickly attach the AED. B) assess for a carotid pulse. C) perform a head-tilt chin-lift. D) begin rescue breathing at once.

Page 7

40. If a pulse cannot be palpated at the radial artery, you should: A) attach an AED at once. B) immediately begin CPR. C) assess the carotid pulse. D) elevate the arm 6 to 12. 41. A patient's refusal for EMS treatment and/or transport must be: A) an informed refusal. B) ignored by the EMT. C) reported to the police. D) witnessed by a notary. 42. After receiving on-line orders from medical control to perform a patient care intervention,

you should: A) perform the intervention as ordered. B) confirm the order in your own words. C) ask the physician to repeat the order. D) repeat the order back word for word.
43. Aspirin is MOST beneficial to patients suspected of having a heart attack because it: A) reduces the associated chest pain. B) dissolves the coronary artery clot. C) causes direct coronary vasodilation. D) prevents the aggregation of platelets. 44. A drug is contraindicated for a patient when it: A) is used to treat a multitude of conditions. B) may cause harm or have no positive effect. C) produces actions other than the desired ones. D) is used to treat a specific medical condition. 45. 40-year-old female is found unconscious by her sister. The patient has a history of

diabetes and has been treated for hypoglycemia in the past. Your glucometer is not functioning. You should: A) place oral glucose between her cheek and gum. B) give oxygen and oral glucose and transport rapidly. C) give oral glucose and carefully monitor her airway. D) administer 100% oxygen and transport her at once.

Page 8

46. At the onset of an acute asthma attack, patients commonly experience difficulty breathing

and: A) audible stridor. B) rales and rhonchi. C) profound cyanosis. D) expiratory wheezing.
47. What is the MOST common cause of airway obstruction in an unconscious patient? A) Vomitus B) The tongue C) Blood clots D) Aspirated fluid 48. The leaf-shaped structure located superior to the larynx is called the: A) epiglottis. B) vallecula. C) cricoid ring. D) thyroid cartilage. 49. A 60-year-old male presents with acute respiratory distress. He is conscious and alert, has

pink and dry skin, and respirations of 24 breaths/min with adequate depth. Which of the following treatment modalities is MOST appropriate for this patient? A) Assisted ventilation with a BVM device and a rapid medical assessment B) Oxygen via a nonrebreathing mask and a focused history and physical exam C) Positive-pressure ventilations and immediate transport to the closest hospital D) Oxygen via a nasal cannula, vital signs, and prompt transport to the hospital
50. A 30-year-old male presents with acute shortness of breath, widespread hives, and facial

swelling. He denies any past medical history and takes no medications. During your assessment, you hear wheezing over all lung fields. His blood pressure is 90/50 mm Hg and his heart rate is 110 beats/min. In addition to giving him 100% oxygen, the MOST important treatment for this patient is: A) albuterol. B) epinephrine. C) an antihistamine. D) a beta-antagonist.

Page 9

51. 55-year-old female with history of emphysema and congestive heart failure. As you assess

her, you notice that she is cyanotic and has severely labored respirations. In between her broken sentences, she states that she has a prescribed inhaler. You should: A) assist ventilations and contact medical control for permission to assist her B) administer the medication, obtain a set of vital signs and document them. C) apply a nonrebreathing mask and administer her inhaler. D) ask the patient when she last used her prescribed inhaler.
52. An acute myocardial infarction occurs when: A) myocardial tissue dies secondary to an absence of oxygen. B) the heart muscle progressively weakens and dysfunctions. C) coronary artery dilation decreases blood flow to the heart. D) the entire left ventricle is damaged and cannot pump blood. 53. Patients with cardiac compromise MOST commonly experience syncope as a result of: A) a cardiac arrhythmia. B) acute pulmonary edema. C) increased cardiac output. D) the associated severe pain. 54. When documenting a patient's description of his or her chest pain or discomfort, it is

MOST important to: A) use medical terminology. B) use the patient's own words. C) underline the patient's quotes. D) document your own perception.
55. Nitroglycerin relieves cardiac-related chest pain by: A) dilating the coronary arteries and improving cardiac blood flow. B) increasing the amount of work that is placed on the myocardium. C) contracting the smooth muscle of the coronary and cerebral arteries. D) constricting the coronary arteries and improving cardiac blood flow. 56. Nitroglycerin should NOT be administered to a patient who: A) has chest discomfort that is unrelieved by rest. B) took a maximum of two doses prior to EMS arrival. C) has a history of medication-controlled hypertension. D) has a systolic blood pressure of less than 100 mm Hg.

Page 10

57. You respond to an office building to find a middle-aged male in cardiac arrest. Coworkers

are performing CPR and state that the patient has been down for approximately 10 minutes. You attach the AED, push the analyze button, and receive a shock advised message. You should A) ensure that nobody is touching the patient and then defibrillate. B) continue CPR for 1 minute and reanalyze the patient's rhythm. C) transport immediately and request a paramedic unit rendezvous. D) call medical control and request permission to stop resuscitation.
58. A 49-year-old male presents with an acute onset of crushing chest pain and diaphoresis.

You should first: A) administer up to 324 mg of baby aspirin. B) administer up to three doses of nitroglycerin. C) obtain vital signs and a SAMPLE history. D) assess the adequacy of his respirations.
59. When assessing a patient with right lower quadrant pain, you should first palpate the: A) left upper quadrant. B) left lower quadrant. C) right upper quadrant. D) right lower quadrant. 60. Dyspnea is MOST accurately defined as: A) shortness of breath or difficulty breathing. B) absence of breath or not breathing. C) a marked increase in the exhalation phase. D) labored breathing with reduced tidal volume. 61. Which of the following statements regarding anaphylaxis is MOST correct? A) Patients with asthma are at lower risk of developing anaphylaxis. B) Anaphylaxis is characterized by airway swelling and hypotension. C) Most anaphylactic reactions occur within 60 minutes after exposure. D) The signs of anaphylaxis are caused by widespread vasoconstriction. 62. The MOST appropriate treatment for a patient with severe abdominal pain and signs of

shock includes: A) administering oxygen via a nasal cannula. B) giving oral fluids to maintain perfusion. C) transporting the patient without delay. D) positioning the patient on his or her side.
Page 11

63. Pain that is localized to the lower back and/or lower abdominal quadrants is MOST

suggestive of: A) acute pancreatitis. B) an aortic aneurysm. C) a kidney infection. D) acute appendicitis.
64. A 19-year-old male complains of not feeling right. His insulin and a syringe are on a

nearby table. The patient says he thinks he took his insulin and can't remember whether or not he ate. He is also unable to tell you the time or what day it is. In addition to administering oxygen, you should: A) administer oral glucose. B) assist him with his insulin injection and reassess him. C) request ALS to administer IV glucose. D) transport and repeat vital signs every 5 minutes en route.
65. Diabetic ketoacidosis occurs when: A) blood glucose levels rapidly fall. B) the cells rapidly metabolize glucose. C) the pancreas produces excess insulin. D) insulin is not available in the body. 66. Classic signs and symptoms of hypoglycemia include: A) warm, dry skin; irritability; bradycardia; rapid respirations. B) cool, clammy skin; weakness; tachycardia; rapid respirations. C) warm, dry skin; hunger; abdominal pain; deep, slow respirations. D) cold, clammy skin; bradycardia; hunger; deep, rapid respirations. 67. Normal blood glucose levels, as measured by a glucometer, are: A) 60-80 mg/dL. B) 80-120 mg/dL. C) 130-150 mg/dL. D) 160-200 mg/dL.

Page 12

68. A 75-year-old male with a history of insulin-dependent diabetes presents with chest pain

and a general feeling of weakness. He tells you that he took his insulin today and ate a regular meal approximately 2 hours ago. You should treat this patient as though he is experiencing: A) hypoglycemia. B) an acute stroke. C) hyperglycemia. D) a heart attack.
69. Urticaria is the medical term for: A) hives. B) burning. C) swelling. D) a wheal. 70. Which of the following physiologic actions does epinephrine produce when given for an

allergic reaction? A) Bronchodilation and vasodilation B) Vasoconstriction and bronchodilation C) Bronchoconstriction and vasoconstriction D) Inhibition of further histamine release
71. When administering epinephrine via auto-injector, you should hold the injector in place for: A) 5 seconds. B) 10 seconds. C) 15 seconds. D) 20 seconds. 72. A 19-year-old female was stung multiple times on the legs by fire ants. She states that she

is allergic to fire ants and has her epi-pen. The patient is conscious and alert and complains of pain to the area of the bites. Upon inspection, you note moderate hives on her lower legs bilaterally. Her blood pressure is 122/70 mm Hg, pulse is 100 beats/min and strong, and respirations are 18 breaths/min and unlabored. You should: A) call medical control for permission to assist her with her epi-pen. B) administer oxygen and transport her to the hospital. C) request a paramedic unit to administer epinephrine. D) apply ice and treat for shock

Page 13

73. The two MOST common signs of anaphylaxis are: A) wheezing and diffuse urticaria. B) watery eyes and localized itching. C) expiratory stridor and tachycardia. D) hypertension and swollen hands. 74. All of the following drugs will cause central nervous system (CNS) depression, EXCEPT: A) opiates. B) amphetamines. C) barbiturates. D) sedative-hypnotics. 75. You respond to a local motel for a young female who was sexually assaulted. The patient

is conscious but confused. She tells you that the last thing she remembers was drinking beer at a club with her friends the night before. When she awoke, she was in the bed of the motel room. You should be MOST suspicious that this patient: A) is a heroin abuser. B) is acutely intoxicated. C) was given Rohypnol. D) is abusing marijuana.
76. During your assessment of a 50-year-old male who was found unconscious in an alley, you

note that he has slow, shallow respirations; significant bradycardia; perioral cyanosis; and pinpoint pupils. As your partner begins assisting the patient's ventilations, he directs your attention to the patient's arms, which have multiple needle tracks on them. This patient's clinical presentation is MOST consistent with: A) a heroin overdose. B) alcohol intoxication. C) a closed head injury. D) an overdose of Valium.
77. A poison that enters the body by ____________ is the MOST difficult to treat. A) Injection B) Ingestion C) Inhalation D) Absorption

Page 14

78. The body's natural cooling mechanism, in which sweat is converted to a gas, is called: A) radiation. B) convection. C) conduction. D) evaporation. 79. Hypothermia occurs when the core body temperature falls below: A) 98F. B) 95F. C) 90F. D) 88F. 80. A person's ability to shiver is lost when his or her body temperature falls below: A) 90F. B) 92F. C) 95F. D) 97F. 81. To obtain the MOST accurate reading of a patient's core body temperature, you should

place a special hypothermia thermometer: A) into the patient's rectum. B) under the patient's tongue. C) behind the patient's knee. D) under the patient's armpit.
82. An air embolism associated with diving occurs when: A) the diver hyperventilates prior to entering the water. B) the diver holds his or her breath during a rapid ascent. C) the alveoli completely collapse due to high pressure. D) high water pressure forces air into the mediastinum. 83. Drowning is MOST accurately defined as: A) temporary survival after submersion in water. B) death from suffocation after submersion in water. C) water in the lungs following submersion in water. D) death beyond 24 hours after submersion in water.

Page 15

84. You and your partner are standing by at a large social event at a river resort when a frantic

woman tells you that she found a young male floating face-down in the water. Nobody claims to have witnessed the event. After you and your partner enter the water and reach the patient, you should: A) protect his spine as you roll him into a supine position. B) begin ventilations with a barrier device or BVM. C) immediately secure him to a longboard with a collar and headblocks. D) perform a jaw-thrust to open his airway and insert an oral airway.
85. You receive a call to a local gymnasium for a basketball player with a dislocated shoulder.

Upon arrival, you find the patient, a 17-year-old male, sitting on the ground. He is holding his left arm in a fixed position away from his body. There is an obvious anterior bulge to the area of injury. You should FIRST: A) assess distal pulse, motor, and sensory functions. B) gently attempt to move his arm toward his body. C) place a pillow under his arm and apply a swathe. D) flex his arm at the elbow and then apply a sling.
86. The central nervous system (CNS) consists of the: A) cerebellum and brain. B) brain and spinal cord. C) cerebrum and meninges. D) meninges and spinal cord. 87. A reflex arc occurs when: A) the motor nerves function automatically without receiving a message from the central

nervous system.
B) a sensory nerve sends a message to the brain but the motor nerve fails to send the

appropriate response to the body. C) the brain interprets sensory information that it receives from peripheral and cranial nerves and sends a signal to the muscles. D) a sensory nerve detects an irritating stimulus and bypasses the brain by sending a direct message to the motor nerve.
88. The five sections of the spinal column, in descending order, are the: A) thoracic, cervical, lumbar, coccygeal, and sacral. B) cervical, thoracic, lumbar, sacral, and coccygeal. C) coccygeal, sacral, lumbar, thoracic, and cervical. D) cervical, coccygeal, thoracic, sacral, and lumbar.

Page 16

89. Any unconscious trauma patient should be assumed to have: A) a history of diabetes mellitus. B) an accompanying spinal injury. C) a severe intracranial hemorrhage. D) internal bleeding in the abdomen. 90. A young male was assaulted and is found unconscious. Your assessment reveals bruising

around his eyes and blood-tinged fluid draining from his nose. You should be MOST suspicious for a/an: A) skull fracture. B) cerebral contusion. C) subdural hematoma. D) intracerebral hemorrhage.
91. During your initial assessment of a 19-year-old unconscious male who experienced severe

head trauma, you note that his respirations are rapid, irregular, and shallow. He has copious ("a very large volume of") bloody secretions flowing from his mouth and nose. You should in order: A) assist his ventilations with a BVM and then suction his oropharynx for up to 15 seconds. B) suction his oropharynx for up to 15 seconds and then assist his ventilations with a BVM. C) immobilize his spine, transport at once, and apply non-rebreather mask. D) pack his nostrils to stop the drainage of blood and call for a helicopter.
92. If you do not have the appropriate size cervical collar, you should: A) use rolled towels or a blanket to immobilize the patient's head and neck.. B) place sandbags on either side of the patient's head. C) ask the patient to keep his or her head in a neutral position. D) use just the lateral head blocks to maintain c-spine alignment.. 93. The MOST common and serious complication of a significant head injury is: A) skull fracture. B) cerebral edema. C) hypoxic seizures. D) acute hypotension.

Page 17

94. Approximately 12 hours after scuba diving with her friends, a 29-year-old female presents

with pain in her elbows and knees. She is conscious and alert and is breathing with adequate tidal volume. When asked, she states that she may have ascended too rapidly during her dive, but didn't experience any symptoms until now. When treating this patient, you should: A) position her supine with her legs elevated 6 and her head down. B) place her in a left lateral recumbent position with her head down. C) administer supplemental oxygen via a nasal cannula at 2 to 4 L/min. D) provide ventilatory assistance with a BVM device and 100% oxygen.
95. In contrast to a behavioral crisis, a psychiatric emergency occurs when a person: A) demonstrates agitation or violence or becomes a threat to self or others. B) experiences feelings of sadness and despair for greater than one month. C) exhibits impaired functioning due to a chemical or genetic disturbance. D) experiences a sudden attack of panic secondary to a stressful situation. 96. It is MOST important for the EMT-B to remember that suicidal patients may: A) inject illicit drugs. B) be self-destructive. C) be homicidal as well. D) have a definitive plan. 97. A 22-year-old male with a history of clinical depression called 9-1-1 and stated that he has

attempted to kill himself. Your unit and law enforcement officers arrive at the scene simultaneously. You find the patient lying supine on the living room floor. He is unconscious, cyanotic, and has snoring respirations. An empty bottle of Dilaudid is found on an adjacent table. You should: A) open the patient's airway and assess his respirations. B) ask the police to handcuff the patient for safety purposes. C) provide care after determining what Dilaudid is used for. D) wait for the police to assess him before providing care.
98. A 49-year-old male presents with confusion, sweating, and visual hallucinations. The

patient's wife tells you that he is a heavy drinker and she thinks he had a seizure shortly before your arrival. This patient is MOST likely experiencing: A) acute hypovolemia. B) alcohol intoxication. C) acute schizophrenia. D) delirium tremens (DTs).

Page 18

99. A 66-year-old male presents with bizarre behavior. His daughter states that he didn't seem

to recognize her and was very rude to her. The patient is conscious, and has a patent airway and adequate breathing. You should: A) conclude that the patient has Alzheimer's disease. B) ask the daughter how her father normally behaves. C) carefully restrain the patient and transport at once. D) advise the patient that his behavior is unacceptable.
100. The disruption of a joint in which the bone' ends are no longer in contact is called a: A) strain. B) sprain. C) fracture. D) dislocation. 101. The primary purpose for splinting a musculoskeletal injury is to: A) prevent further injury. B) maximize distal circulation. C) make the patient comfortable. D) facilitate ambulance transport. 102. Prior to splinting an open extremity fracture, the EMT-B should first: A) assess the injury for crepitation. B) manually stabilize the injury site. C) assess distal neurovascular status. D) cover all bleeding or open wounds. 103. A 22-year-old female was ejected from her car after striking a tree head-on. As you

approach her, you note obvious closed deformities to both of her femurs. She is not moving and does not appear to be conscious. You should: A) apply manual stabilization to both of her femurs. B) administer oxygen and perform a rapid assessment. C) assess for a carotid pulse and assist her ventilations. D) stabilize her head and perform an initial assessment.

Page 19

104. A 17-year-old football player collided with another player and has pain to his left

clavicular area. He is holding his arm against his chest and refuses to move it. Your assessment reveals obvious deformity to the midshaft clavicle. After assessing distal circulation and neurological function, you should: A) perform a detailed physical examination. B) straighten his arm and apply a board splint. C) immobilize the injury with a sling and swathe. D) place a pillow under his arm and apply a sling.
105. Bonus #1: Label up to 10 bones of the skeleton. (5 points total on this item) 106. Bonus #2: Sketch the abdominal cavity, divide it and label the quadrants and draw and label up to 6 organs in it. (5 points total on this item) 107. Bonus #3. List the steps in a medical or trauma patient assessment. (5 points total on this item)

Page 20

Answer Key - FHS EMT midyear exam 2011 1. 2. 3. 4. 5. 6. 7. 8. 9. 10. 11. 12. 13. 14. 15. 16. 17. 18. 19. 20. 21. 22. 23. 24. 25. 26. 27. 28. 29. 30. 31. 32. 33. 34. 35. 36. 37. 38. 39. 40. 41. 42. 43. 44. 45. 46. 47. 48. 49. 50. 51. 52. B A A B B A D B D C C A C B C C A B B C A C B D C D B D C D B D A B B B A D C C A D D B D D B A B B A A
Page 21

53. 54. 55. 56. 57. 58. 59. 60. 61. 62. 63. 64. 65. 66. 67. 68. 69. 70. 71. 72. 73. 74. 75. 76. 77. 78. 79. 80. 81. 82. 83. 84. 85. 86. 87. 88. 89. 90. 91. 92. 93. 94. 95. 96. 97. 98. 99. 100. 101. 102. 103. 104. 105. 106.

A B A D A D A A B C B A D B B D A B B B A B C A A D B A A B B A A B D B B A B A B B A C A D B D A D D C

Page 22

107.

Page 23

Name: ___________________________________ ______________

Date:

1. The automated external defibrillator (AED) should be applied to: A) uninjured patients who are pulseless and apneic. B) trauma patients who are in cardiopulmonary arrest. C) medical patients who have a very weak, slow pulse. D) patients who you think may experience cardiac arrest.

2. Complications associated with chest compressions include all of the following, EXCEPT: A) rib fractures. B) liver laceration. C) gastric distention. D) a fractured sternum.

3. Most out-of-hospital cardiac arrests occur as the result of: A) massive blunt trauma. B) a cardiac dysrhythmia. C) an acute ischemic stroke. D) obstruction of the airway.

4. Basic life support (BLS) is noninvasive, meaning that it: A) can only be performed by EMT-Basics. B) focuses only on severe bleeding control. C) is only used to treat an obstructed airway. D) does not involve penetration of the body.

5. You should deliver chest compressions to an unconscious adult patient in cardiac arrest by: A) compressing quickly and releasing slowly. B) compressing the lower third of the sternum. C) placing the heel of your hand on the xiphoid. D) depressing the sternum 2 inches to 2 ? inches.

Page 1

6. The person who is responsible for authorizing EMT-B personnel to perform emergency medical care is the: A) shift supervisor. B) medical director. C) lead EMT in charge. D) EMS administrator.

7. As an EMT-B, you may be authorized to administer aspirin to a patient with chest pain based on: A) the patient's condition. B) your local EMS protocols. C) the national standard of care. D) transport time to the hospital.

8. Obtaining continuing medical education is the responsibility of the: A) State Bureau of EMS. B) EMS training director. C) EMS medical director. D) individual EMT-B.

9. Which of the following statements MOST accurately describes emergency medical services (EMS)? A) A vast network of advanced life support (ALS) providers who provide definitive emergency care in the prehospital setting B) A team of healthcare professionals who are responsible for providing emergency care and transportation to the sick and injured C) A system composed exclusively of basic-level EMTs who are responsible for providing care to sick and injured patients D) A team of paramedics and emergency physicians who are responsible for providing emergency care to critically injured patients

Page 2

10. Cardiac monitoring, pharmacological interventions, and other advanced treatment skills are functions of the: A) EMT-Basic. B) EMT-Paramedic. C) EMT-Intermediate. D) certified first responder.

11. What type of stress reaction occurs when an EMT-B is exposed to many insignificant stressors over a period of several months or years? A) Acute stress reaction B) Cumulative stress reaction C) Posttraumatic stress reaction D) Critical incident stress reaction

12. Which of the following statements about communicable diseases is correct? A) Not all people who are infected with a communicable disease show signs or symptoms of illness. B) Anyone who has been exposed to a communicable disease always is a carrier of that disease. C) Anyone who is infected with a communicable disease shows signs or symptoms of that disease. D) Any time you take the temperature of a patient who has a communicable disease, you need to wear gloves or you will become infected.

13. Determination of exposure is an important part of an exposure control plan because it: A) determines the time of day that most exposures are likely to occur. B) determines which type of communicable diseases might be present in the workplace. C) defines who is most likely to transmit communicable diseases in the workplace. D) defines who is at risk for contact with blood and body fluids and which tasks pose a risk of exposure.
Page 3

14. The chemical and physical reactions commonly referred to as the fight or flight response involve interaction of the: A) nervous and endocrine systems. B) respiratory and nervous systems. C) gastrointestinal and nervous systems. D) cardiovascular and endocrine systems.

15. Which of the following statements would NOT be appropriate to say to the family of a dying patient? A) Things will get better in time. B) It is okay to be angry and sad. C) This must be painful for you. D) Tell me how you are feeling.

16. Each pelvic bone is formed by the fusion of the: A) sacrum and ischium. B) ilium, ischium, and pubis. C) pubis and acetabulum. D) ilium, pubis, and sacrum.

17. Worn out blood cells, foreign substances, and bacteria are filtered from the blood by the: A) liver. B) spleen. C) kidney. D) pancreas.

18. The cervical spine is composed of ____ vertebrae. A) 4 B) 5 C) 6 D) 7

Page 4

19. Together, the right and left lungs contain how many lobes? A) 3 B) 4 C) 5 D) 6

20. Voluntary muscle becomes paralyzed when: A) insufficient oxygen is carried to the muscle. B) nerve supply is lost through injury to the brain. C) blood supply to the muscle is markedly decreased. D) acidic waste products accumulate within the muscle.

21. The primary function of the right atrium is to: A) receive blood from the vena cava. B) pump blood to the pulmonary artery. C) receive blood from the pulmonary veins. D) pump blood to the lungs for reoxygenation.

22. A 5-year-old boy has fallen and has a severe deformity of the forearm near the wrist. He has possibly sustained a fracture of the: A) proximal forearm. B) superior forearm. C) dorsal forearm. D) distal forearm.

23. You should assess an infant or child's skin color on his or her: A) arms and legs. B) fingers and toes. C) chest and abdomen. D) palms and soles.

Page 5

24. A patient with spontaneous respirations is breathing: A) at a normal rate. B) with shallow depth. C) without difficulty. D) without assistance.

25. The pressure exerted against the walls of the artery when the left ventricle contracts is called the: A) blood pressure. B) systolic pressure. C) diastolic pressure. D) pulse pressure.

26. Which of the following questions is used to determine a patient's chief complaint? A) "What seems to be the matter?" B) "When did the chest pain begin?" C) "Are you having trouble breathing? D) "Do you have a history of diabetes?

27. Typical methods of assessing a patient's breathing include all of the following, EXCEPT: A) listening to breath sounds with a stethoscope. B) observing for nasal flaring during inhalation. C) observing the chest for adequate rise and fall. D) feeling for air movement at the nose and mouth.

28. An elderly patient has fallen and hit her head. You assess her level of consciousness as unresponsive using the AVPU scale. Your initial care should focus on: A) obtaining baseline vital signs. B) gathering medical history data. C) providing immediate transport. D) airway, breathing, and circulation.

Page 6

29. When assessing the skin of an unconscious patient, you note that it has a bluish tint to it. This finding is called: A) pallor. B) flushing. C) cyanosis. D) mottling.

30. You are attempting to resuscitate a 50-year-old male in cardiac arrest. He is on the second floor of his home. What is the MOST appropriate device to use when carrying him to the first floor? A) Long backboard B) Wheeled stretcher C) Folding stair chair D) Portable stretcher

31. When carrying a patient up or down stairs, you should avoid: A) flexing your body at the knees. B) the use of more than two EMT-Bs. C) using a wheeled stretcher whenever possible. D) the use of a long backboard or scoop stretcher.

32. What is the MOST appropriate method to use when moving a patient from his or her bed to the wheeled stretcher? A) Log roll B) Direct carry C) Extremity carry D) Draw sheet method

Page 7

33. Which of the following statements regarding the one-person bag-valve mask (BVM) device is correct? A) BVM ventilations should be delivered over 2 seconds when the device is being operated by one person. B) The C-clamp method of holding the mask to the face is not effective when ventilating a patient with a BVM. C) Adequate tidal volume is often difficult to achieve when one EMT-B is operating the BVM device. D) The BVM device delivers more tidal volume and a higher oxygen concentration than the mouth-to-mask technique.

34. Structures of the lower airway include all of the following, EXCEPT the: A) larynx. B) trachea. C) pharynx. D) alveoli.

35. The exchange of air between the lungs and the environment is called: A) diffusion. B) ventilation. C) respiration. D) oxygenation.

36. When administering supplemental oxygen to a patient over a prolonged period of time, you should: A) routinely use a nasal cannula. B) place the patient in a supine position. C) set the flow rate to no more than 4 L/min. D) consider attaching an oxygen humidifier.

37. Proper technique for suctioning the oropharynx of an adult patient includes: A) continuously suctioning patients with copious oral secretions. B) suctioning while withdrawing the catheter from the oropharynx. C) removing large, solid objects with a tonsil-tip suction catheter. D) suctioning for up to 1 minute if the patient is well-oxygenated.
Page 8

38. The purpose of the pin-indexing system that has been established for compressed gas cylinders is to: A) ensure that the correct regulator is used for the cylinder. B) help you determine what type of oxygen regulator to use. C) prevent destroying or stripping the threads on the cylinder. D) reduce the cylinder pressure to a safe and more useful range.

39. At a flow rate of 6 liters per minute (L/min), a nasal cannula can deliver an approximate oxygen concentration of ___%. A) 24 B) 35 C) 44 D) 52

40. After performing a head-tilt chin-lift to open the airway of an unconscious patient, you should: A) place him or her in the recovery position. B) provide positive-pressure ventilatory assistance. C) assess respiratory rate, depth, and regularity. D) suction as needed and insert an airway adjunct.

41. In order to facilitate the emergency department physician's diagnosis of a patient's illness, you should: A) document the patient's complaint in his or her own words. B) record your own findings using quotation marks as needed. C) forgo the medical history until the patient is at the hospital. D) document your perception of what is wrong with the patient.

42. Diminished perfusion manifests with skin that is: A) pink. B) yellow. C) red. D) pale.

Page 9

43. A 71-year-old female slipped on a rug and fell. She is conscious and alert and complains of pelvic pain. Her respirations are 22 breaths/min with adequate depth and her heart rate is 110 beats/min. Which of the following would NOT be appropriate for this patient? A) A focused physical exam B) 100% supplemental oxygen C) Treating her for possible shock D) Gentle palpation of the pelvis

44. Observations made when forming a general impression of a patient would include all of the following, EXCEPT: A) appearance. B) pulse strength. C) race and gender. D) level of distress.

45. Which of the following patients does NOT have an altered mental status? A) A patient with an acute allergic reaction and dizziness B) A diabetic who opens his eyes when you ask questions C) A head-injured patient who is slow to answer questions D) A patient who overdosed and moans when he is touched

46. While en route to a call for a patient with chest pain, you encounter a bridge that is covered with fast-moving water and is not barricaded off. You should: A) notify dispatch that you are out of service and have another unit respond. B) contact dispatch to have barricades placed and attempt to cross the bridge. C) notify the dispatcher of your delay and take an alternate route to the scene. D) use your ambulance as a barricade to prevent others from crossing the bridge.

Page 10

47. Information included in a radio report to the receiving hospital should include all of the following, EXCEPT: A) your perception of the severity of the problem. B) a preliminary diagnosis of the patient's problem. C) a brief history of the patient's current problem. D) a brief summary of the care that you provided.

48. General guidelines for effective radio communication include all of the following, EXCEPT: A) limiting your radio transmission to 30 seconds. B) acknowledging a transmission as soon as possible. C) using 10 codes to ensure radio traffic confidentiality. D) holding the microphone about 2 to 3 from your mouth.

49. What medication route is commonly used to deliver emergency drugs to children when intravenous access cannot be obtained? A) Intraosseous B) Intramuscular C) Subcutaneous D) Transcutaneous

50. When gathering a patient's medications, you find the following: Isordil, Lasix, Motrin, and Digoxin. Which of these medications can be obtained over-the-counter (OTC)? A) Lasix B) Motrin C) Isordil D) Digoxin

Page 11

51. A 62-year-old male presents with crushing chest pain, which he describes as being the same kind of pain that he had with a previous heart attack. He has prescribed nitroglycerin, but states that he has not taken any. After administering 100% oxygen and contacting medical control, you should: A) begin immediate transport and request a rendezvous with a paramedic unit. B) assist him with his nitroglycerin unless his systolic BP is less than 100 mm Hg. C) administer up to three doses of nitroglycerin before assessing his blood pressure. D) administer the nitroglycerin unless he has taken Viagra within the past 72 hours.

52. The term pharmacology is MOST accurately defined as: A) the study of drugs that are produced illegally. B) the study of how medications affect the brain. C) the study of drugs and their actions on the body. D) the study of drug excretion from the human body.

53. When auscultating the lungs of a patient with respiratory distress, you hear adventitious sounds. This means that the patient has: A) normal breath sounds. B) abnormal breath sounds. C) diminished breath sounds. D) an absence of breath sounds.

54. A pleural effusion is MOST accurately defined as: A) a unilaterally collapsed lung. B) diffuse collapsing of the alveoli. C) fluid accumulation outside the lung. D) a bacterial infection of the lung tissue.

Page 12

55. In order for efficient pulmonary gas exchange to occur: A) the percentage of inhaled carbon dioxide must exceed the percentage of inhaled oxygen. B) there must be low quantities of pulmonary surfactant to allow for full alveolar expansion. C) the pulmonary capillaries must be completely constricted and the alveoli must be collapsed. D) oxygen and carbon dioxide must be able to freely diffuse across the alveolar-capillary membrane.

56. Which of the following statements regarding the hypoxic drive is MOST correct? A) The hypoxic drive stimulates a person to breathe on the basis of low oxygen levels. B) Chronic carbon dioxide elimination often results in activation of the hypoxic drive. C) The hypoxic drive serves as the primary stimulus for breathing in healthy individuals. D) 100% supplemental oxygen will always cause apnea in patients with a hypoxic drive.

57. In what area of the lungs does respiration occur? A) Alveoli B) Trachea C) Bronchi D) Capillaries

58. Weakening of the airway in patients with chronic bronchitis is the result of: A) destruction of protective mechanisms that remove foreign particles. B) loss of the lubricating substance that facilitates alveolar expansion. C) airway irritation caused by a marked decrease in mucus production. D) acute constriction of the bronchioles caused by an external irritant.
Page 13

59. Major risk factors for a myocardial infarction include all of the following, EXCEPT: A) hypoglycemia. B) hypertension. C) diabetes mellitus. D) elevated cholesterol.

60. Upon arriving at the residence of a patient with a possible cardiac problem, it is MOST important to: A) assess the scene for potential hazards. B) determine if you need additional help. C) request a paramedic unit for assistance. D) gain immediate access to the patient.

61. You are dispatched to a convenience store for a possible cardiac arrest. Upon arriving at the scene, you find two bystanders performing CPR on the patient, a 48-year-old male. Your initial action should be to: A) assess the effectiveness of the bystander's CPR. B) quickly attach the AED and push the analyze button. C) have the bystanders stop CPR and assess the patient. D) request a paramedic unit and quickly attach the AED.

62. Which of the following signs indicates the onset of decompensated cardiogenic shock? A) Restlessness B) Pale, clammy skin C) Low blood pressure D) Significant tachycardia

Page 14

63. Your EMS team is performing CPR on a 60-year-old male in cardiac arrest. You connect the AED, push the analyze button, and receive a no shock advised message. Your MOST appropriate action should be to: A) reanalyze the patient's cardiac rhythm. B) determine if a palpable pulse is present. C) perform CPR and transport immediately. D) immediately assess the patient's airway.

64. You respond to an office building to find a middle-aged male in cardiac arrest. Coworkers are performing CPR and state that the patient has been down for approximately 10 minutes. You attach the AED, push the analyze button, and receive a shock advised message. You should A) ensure that nobody is touching the patient and then defibrillate. B) continue CPR for 1 minute and reanalyze the patient's rhythm. C) transport immediately and request a paramedic unit rendezvous. D) call medical control and request permission to stop resuscitation.

65. The EMT-B should use an automated external defibrillator (AED) on a child over 1 year of age if: A) he or she is not breathing and has a weakly palpable pulse. B) his or her condition is rapidly progressing to cardiac arrest. C) pediatric pads and an energy reducing device are available. D) special pads are used and the child has profound tachycardia.

66. Muscle control and body coordination are controlled by the: A) cerebrum. B) cerebellum. C) brain stem. D) cerebral cortex.

67. The left cerebral hemisphere controls: A) the right side of the face. B) heart rate and pupil reaction. C) the right side of the body. D) breathing and blood pressure.
Page 15

68. When assessing arm movement of a patient with a suspected stroke, you should: A) observe for arm movement for approximately 5 minutes. B) expect to see one arm slowly drift down to the patient's side. C) ask the patient to close his or her eyes during the assessment. D) ask the patient to hold his or her arms up with the palms down.

69. A transient ischemic attack (TIA) occurs when: A) medications are given to dissolve a cerebral blood clot. B) a small cerebral artery ruptures and causes minimal damage. C) the normal body processes destroy a clot in a cerebral artery. D) signs and symptoms resolve spontaneously within 48 hours.

70. When caring for a child with documented hypoglycemia, you should be MOST alert for: A) a seizure. B) an acute stroke. C) respiratory distress. D) a febrile convulsion.

71. The anterior aspect of the cerebrum controls: A) touch. B) emotion. C) movement. D) vision.

72. The parietal peritoneum lines the: A) retroperitoneal space. B) lungs and chest cavity. C) walls of the abdominal cavity. D) surface of the abdominal organs.

Page 16

73. You are transporting a 49-year-old male with tearing abdominal pain. You are approximately 30 miles away from the closest hospital. During your ongoing assessment, you determine that the patient's condition has deteriorated significantly. You should: A) begin ventilatory assistance with a BVM device. B) immediately perform a detailed physical examination. C) continue transporting and alert the receiving hospital. D) consider requesting a rendezvous with an ALS unit.

74. Common signs and symptoms of acute abdomen include all of the following, EXCEPT: A) tachycardia. B) hypertension. C) nausea and vomiting. D) abdominal guarding.

75. Colic is pain caused by: A) irritation of the parietal peritoneum. B) distention or contraction of hollow organs. C) blunt trauma to the solid abdominal organs. D) vigorous palpation of the abdominal quadrants.

76. A 47-year-old male presents with severe abdominal pain of 3 hours' duration. His abdomen is distended and guarded. Your MOST important consideration for this patient should be to: A) transport him in a supine position. B) be alert for signs and symptoms of shock. C) assess his BP to determine perfusion adequacy. D) determine the exact location and cause of his pain.

Page 17

77. Which of the following statements regarding an ectopic pregnancy is correct? A) Signs of an ectopic pregnancy usually present within the first 6 to 8 weeks of pregnancy. B) The abdominal cavity is the most common location for abnormal implantation of a fertilized egg. C) An ectopic pregnancy almost always presents with acute signs and symptoms in the third trimester. D) Pain from a ruptured ectopic pregnancy is usually severe but the bleeding is typically rather minimal.

78. Which of the following conditions is the diabetic patient at an increased risk of developing? A) Blindness B) Depression C) Alcoholism D) Hepatitis B

79. Which of the following statements regarding diabetic coma is correct? A) Diabetic coma can be prevented by taking smaller insulin doses. B) Diabetic coma typically develops over a period of hours or days. C) Patients with low blood glucose levels are prone to diabetic coma. D) Diabetic coma rapidly progresses once hyperglycemia develops.

80. Patients with type II diabetes usually control their disease with all of the following, EXCEPT: A) diet and exercise. B) tolbutamide (Orinase). C) glyburide (Micronase). D) supplemental insulin.

Page 18

81. You respond to a movie theater for a 70-year-old male who is confused. His wife tells you he has type II diabetes but refuses to take his pills. Your assessment reveals that the patient is diaphoretic, tachycardic, and tachypneic. Initial management for this patient should include: A) administering 1 to 2 tubes of oral glucose. B) applying a nonrebreathing mask at 15 L/min. C) assisting the patient with his diabetic medication. D) performing a rapid exam and obtaining vital signs.

82. Type I diabetes: A) is typically treated with medications such as Diabenase. B) is a condition in which no insulin is produced by the body. C) typically occurs in patients between 50 and 70 years of age. D) is defined as a blood sugar level that is less than 120 mg/dL.

83. Diabetic ketoacidosis occurs when: A) blood glucose levels rapidly fall. B) the cells rapidly metabolize glucose. C) the pancreas produces excess insulin. D) insulin is not available in the body.

84. Most patients who die of anaphylaxis do so within the first: A) 5 minutes following exposure. B) 30 minutes following exposure. C) 60 minutes following exposure. D) 90 minutes following exposure.

85. Which of the following patients would MOST likely have a delayed onset of an allergic reaction? A) A 21-year-old female who inhaled pollen B) A 30-year-old male who was stung by a bee C) A 45-year-old male who ingested penicillin D) A 50-year-old male who was exposed to latex

Page 19

86. A 50-year-old male was stung by a honeybee approximately 15 minutes ago. He presents with respiratory distress, facial swelling, and hypotension. After placing him on oxygen and administering his epinephrine via auto-injector, you note that his breathing has improved. Additionally, his facial swelling is resolving and his blood pressure is stable. Your next action should be to: A) load him into the ambulance and transport to the hospital. B) visualize his airway to assess for oropharyngeal swelling. C) notify medical control of the situation and your treatment. D) record the time and dose of the injection on the run form.

87. Which of the following adventitious sounds indicates swelling of the upper airway? A) Rales B) Stridor C) Rhonchi D) Wheezing

88. A 38-year-old female was bitten by fire ants while at the park with her kids. Your initial assessment reveals that she is semiconscious, has profoundly labored breathing with reduced tidal volume, and has a rapid, thready pulse. She has a red rash on her entire body and her face appears swollen. You should: A) administer 0.3 mg of epinephrine 1:1,000. B) assist her ventilations with 100% oxygen. C) apply oxygen and perform a rapid assessment. D) place her supine with her legs elevated 6 to 12.

89. A raised, swollen, well-defined area on the skin that is the result of an insect bite or sting is called: A) hives. B) purpura. C) urticaria. D) a wheal.

Page 20

90. You have administered one dose of epinephrine to a 40-year-old female to treat an allergic reaction that she developed after being stung by a scorpion. Your reassessment reveals that she is still having difficulty breathing, has a decreasing mental status, and has a blood pressure of 80/50 mm Hg. You should: A) monitor her en route to the hospital and call medical control if she worsens. B) crush up an antihistamine tablet and place it in between her cheek and gum. C) request permission from medical control to give another dose of epinephrine. D) administer a nebulized bronchodilator to improve the status of her breathing.

91. You respond to a college campus for a young male who is acting strange. After law enforcement has secured the scene, you enter the patient's dorm room and find him sitting on the edge of the bed; he appears agitated. As you approach him, you note that he has dried blood around both nostrils. He is breathing adequately, has a rapid, irregular pulse, and his BP is 200/110 mm Hg. Treatment for this patient includes: A) requesting a paramedic to administer naloxone (Narcan). B) assisting his ventilations with a BVM and 100% oxygen. C) asking law enforcement to place handcuffs on the patient. D) attempting to calm him and giving him oxygen if tolerated.

92. Which of the following drugs is NOT a sedative-hypnotic? A) Seconal B) Valium C) Cocaine D) Rohypnol

93. The MOST appropriate drug to use when diluting an airborne substance is: A) oxygen. B) syrup of ipecac. C) activated charcoal. D) an alkaline antidote.

Page 21

94. As you enter the residence of a patient who has possibly overdosed, you should: A) be alert for personal hazards. B) look for drug paraphernalia. C) observe the scene for drug bottles. D) quickly gain access to the patient.

95. During your assessment of a 50-year-old male who was found unconscious in an alley, you note that he has slow, shallow respirations; significant bradycardia; perioral cyanosis; and pinpoint pupils. As your partner begins assisting the patient's ventilations, he directs your attention to the patient's arms, which have multiple needle tracks on them. This patient's clinical presentation is MOST consistent with: A) a heroin overdose. B) alcohol intoxication. C) a closed head injury. D) an overdose of Valium.

96. When the body loses sweat, it also loses: A) plasma. B) nutrients. C) erythrocytes. D) electrolytes.

97. The diving reflex may allow a person to survive extended periods of submersion in cold water secondary to: A) bradycardia and a slowing of the metabolic rate. B) laryngospasm that protects the lungs from water. C) tachycardia and a lowering of the blood pressure. D) increases in the metabolic rate and oxygen demand.

Page 22

98. A 48-year-old male was stung on the leg by a jellyfish while swimming in the ocean. He is conscious and alert, but complains of intense pain at the wound site. Specific treatment for this patient includes: A) irrigating the wound with vinegar and immersing his leg in hot water. B) pulling the nematocysts out with tweezers and bandaging the wound. C) immersing his leg in fresh cold water and scraping away the stingers. D) applying a chemical ice pack to the wound and encouraging movement.

99. The MOST prominent symptom of decompression sickness is: A) tightness in the chest. B) difficulty with vision. C) dizziness and nausea. D) abdominal or joint pain.

100. A 20-year-old male was pulled from cold water by his friends. The length of his submersion is not known and was not witnessed. You perform an initial assessment and determine that the patient is apneic and has a slow, weak pulse. You should: A) suction his airway for 30 seconds, provide rescue breathing, keep him warm, and transport at once. B) ventilate with a BVM device, apply a cervical collar, remove his wet clothing, and transport rapidly. C) provide rescue breathing, remove wet clothing, immobilize his spine, keep him warm, and transport carefully. D) apply 100% oxygen via a nonrebreathing mask, immobilize his spine, keep him warm, and transport rapidly.

Page 23

101. A 78-year-old female presents with an acute change in her behavior. The patient's son tells you that his mother has a history of type II diabetes and was diagnosed with Alzheimer's disease 6 months ago. The patient's speech is slurred and she is not alert to her surroundings. You should: A) transport the patient to a psychiatric facility. B) inquire about the possibility of head trauma. C) conclude that the patient's blood sugar is high. D) allow the patient to refuse transport if she wishes.

102. In contrast to a behavioral crisis, a psychiatric emergency occurs when a person: A) demonstrates agitation or violence or becomes a threat to self or others. B) experiences feelings of sadness and despair for greater than one month. C) exhibits impaired functioning due to a chemical or genetic disturbance. D) experiences a sudden attack of panic secondary to a stressful situation.

103. The single most significant factor that contributes to suicide is: A) depression. B) drug abuse. C) advanced age. D) a chronic illness.

104. From a mental health standpoint, an abnormal or disturbing pattern of behavior is a matter of concern if it lasts for at least: A) 1 week. B) 2 weeks. C) 3 weeks. D) 4 weeks.

Page 24

105. Following delivery of a full-term baby, you have properly cared for the baby and have clamped and cut the umbilical cord. During transport, you note that the mother is experiencing moderate vaginal bleeding. You should: A) elevate her legs 6 to 8 and cover her with a blanket. B) firmly massage the uterine fundus with a circular motion. C) carefully insert a sterile trauma dressing into her vagina. D) place her legs together and position her on her left side.

106. A placenta previa is MOST accurately defined as: A) delivery of a portion of the placenta before the baby. B) premature placental separation from the uterine wall. C) abnormal development and functioning of the placenta. D) development of the placenta over the cervical opening.

107. When the mother is experiencing a contraction, you should instruct her to: A) hold her breath. B) push for 30 seconds. C) take quick short breaths. D) rest and breathe deeply.

108. You are assessing a 30-year-old woman who is 35 weeks' pregnant. She tells you that her amniotic sac has not ruptured, but she is experiencing irregular contractions that come and go. Upon visual inspection, you note a small amount of brown mucus draining from her vagina. You should: A) allow her to drive to the hospital. B) administer oxygen and transport. C) prepare for an emergency delivery. D) perform a rapid medical assessment.

Page 25

109. A 30-year-old pregnant female is gravida-3 and para-2. This means that she has: A) given birth to 2 live babies. B) given birth to 5 live babies. C) had 3 miscarriages in the past. D) been pregnant a total of 3 times.

110. Which of the following patients has experienced the MOST significant fall? A) 4'8 tall patient who fell 13 feet B) 5'0 tall patient who fell 13 feet C) 4'6 tall patient who fell 13 feet D) 5'9 tall patient who fell 14 feet

111. Which of the following statements regarding gunshot wounds is MOST correct? A) High velocity bullets will cause less severe internal injuries. B) The size of a bullet has the greatest impact on producing injury. C) Low velocity bullets will cause the greatest amount of trauma. D) The speed of a bullet has the greatest impact on producing injury.

112. The cervical spine is MOST protected from whiplash-type injuries when the: A) headrest is appropriately positioned. B) airbag correctly deploys upon impact. C) patient tenses up at the time of impact. D) rear end of the vehicle is initially struck.

113. Two of the MOST common mechanisms of injury for blunt trauma are: A) falls and motor-vehicle collisions. B) low-caliber gunshot wounds and falls. C) gunshot wounds and vehicle ejections. D) motor-vehicle collisions and stabbings.

Page 26

114. Which of the following findings would be LEAST suggestive of the presence of high-energy trauma? A) Dismounted seats B) Steering wheel deformity C) Intrusion into the vehicle D) Deployment of the airbag

115. Blood stasis, changes in the vessel wall, and certain medications affect the: A) systolic blood pressure exclusively. B) ability of the blood to effectively clot. C) ability of red blood cells to carry oxygen. D) white blood cell's ability to fight infection.

116. The pneumatic antishock garment (PASG) would MOST likely be effective in: A) forcing blood from the lungs of a patient with pulmonary edema. B) maintaining adequate perfusion to the fetus during the third trimester. C) controlling severe external bleeding associated with pelvic fractures. D) controlling severe intrathoracic bleeding caused by penetrating trauma.

117. Which of the following statements regarding the clotting of blood is MOST correct? A) Venous and capillary blood typically does not clot spontaneously. B) Bleeding begins to clot when the end of a damaged vessel dilates. C) Direct contact with the environment prevents blood from clotting. D) A person taking aspirin will experience slower blood clotting time.

Page 27

118. Which of the following organs can tolerate inadequate perfusion for up to 2 hours? A) Brain B) Heart C) Kidneys D) Muscles

119. You arrive at the home of a 50-year-old female with severe epistaxis. As you are treating her, it is MOST important to recall that: A) the patient may be significantly hypertensive. B) the patient is at risk for vomiting and aspiration. C) a detailed exam is needed to determine the cause. D) many medications interfere with blood clotting.

120. The systemic veins function by: A) returning deoxygenated blood back to the heart. B) delivering oxygen-poor blood to the capillaries. C) returning oxygen-rich blood back to the left atrium. D) delivering oxygen-rich blood to the capillaries.

121. After applying a dressing to an arterial bleed from a patient's arm, you notice that the dressing quickly becomes soaked with blood. You should: A) replace the dressing with another dressing. B) apply a tourniquet proximal to the wound. C) splint the arm and keep it below heart level. D) place additional dressings over the wound.

122. When the body senses a state of hypoperfusion, the sympathetic nervous system releases epinephrine, the effects of which include: A) tachypnea. B) tachycardia. C) vasodilation. D) restlessness.

Page 28

123. Capillary sphincters are: A) under complete control of the voluntary portion of the nervous system. B) capable of dilating in order to increase perfusion to crucial body organs. C) responsible for constricting to compensate for decreased cell perfusion. D) circular muscular walls that regulate blood flow through the capillaries.

124. Neurogenic shock occurs when: A) failure of the nervous system causes widespread vasodilation. B) the spinal cord is severed and causes massive hemorrhaging. C) there is too much blood to fill a smaller vascular container. D) massive vasoconstriction occurs distal to a spinal cord injury.

125. A 19-year-old male was stung multiple times by fire ants. He is experiencing obvious signs and symptoms of anaphylactic shock. You administer 100% oxygen and give him epinephrine via subcutaneous injection. Upon reassessment, you determine that his condition has not improved. You should: A) transport him immediately and provide supportive care while en route. B) consider that he may actually be experiencing an acute asthma attack. C) repeat the epinephrine injection after consulting with medical control. D) request a paramedic unit that is stationed approximately 15 miles away.

126. A 70-year-old female was recently discharged from the hospital following a total hip replacement. Today, she presents with an altered mental status, tachycardia, and a blood pressure of 100/64 mm Hg. Her skin is warm and moist. You should be MOST suspicious that she is experiencing: A) septic shock. B) pump failure. C) a local infection. D) decompensated shock.
Page 29

127. You are dispatched to a residence for a 40-year-old female who fainted. Upon your arrival, the patient is conscious and alert and states that she is fine. Her husband tells you that she fainted after receiving news that her sister was killed in a car crash. You offer oxygen to the patient, but she refuses to accept it. At this point, your primary concern should be to: A) determine if she was injured when she fainted. B) provide emotional support regarding her sister. C) advise her that she needs to go to the hospital. D) obtain baseline vital signs and a medical history.

128. Patients develop septic shock secondary to: A) poor vessel function and severe volume loss. B) an infection that weakens cardiac contractions. C) failure of the blood vessels to adequately dilate. D) weak vessel tone due to nervous system damage.

129. All of the following body structures are lined with mucous membranes, EXCEPT for the: A) lips. B) nose. C) anus. D) mouth.

130. When assessing a patient with a closed soft tissue injury, it is MOST important to: A) manipulate the injury site for signs of a fracture. B) assess circulation distal to the site of the injury. C) remain alert for more severe underlying injuries. D) recognize that the integrity of the skin is broken.

131. When a person is exposed to a cold environment: A) sweat is produced and is warmed when the vessels constrict. B) blood vessels dilate and divert blood to the core of the body. C) the skin becomes flushed secondary to peripheral vasodilation. D) peripheral vessels constrict and divert blood away from the skin.

Page 30

132. You and your partner arrive at the scene of a house fire where fire fighters have rescued a 50-year-old male from his burning house. The patient has superficial and partialthickness burns to his face and chest. His nasal hairs are singed and he is coughing up sooty sputum. You should be MOST concerned with: A) treating him for hypothermia. B) preventing the risk of infection. C) estimating the extent of his burns. D) the potential for airway swelling.

133. An A) B) C) D)

abdominal evisceration: is most commonly the result of blunt force trauma. should be covered with bulky dry, sterile dressings. often causes severe hypothermia because of heat loss. occurs when organs protrude through an open wound.

134. Burns are MOST appropriately classified according to: A) depth and extent. B) location and pain. C) degree and location. D) extent and location.

135. In contrast to animal bites, the bite of a human: A) is usually less severe because the human mouth is cleaner. B) typically results in a minor infection that is slow-spreading. C) carries with it a wide variety of virulent bacteria and viruses. D) is associated with a much higher incidence of rabies infection.

136. The eyeball itself is referred to as the: A) orbit. B) globe. C) sclera. D) cornea.

Page 31

137. Which of the following statements regarding the vitreous humor is MOST correct? A) Vitreous humor is a clear, watery fluid that cannot be replaced if it is lost during an eye injury. B) Vitreous humor is a clear, jellylike fluid near the back of the eye that cannot be replaced if it is lost. C) Vitreous humor is a clear, watery fluid that is located in front of the lens and can be replaced if it is lost. D) Vitreous humor is a clear fluid that is produced by the lacrimal glands and cannot be replaced if it is lost.

138. The light-sensitive area of the eye where images are projected is called the: A) iris. B) lens. C) retina. D) conjunctiva.

139. The term hyphema is MOST accurately defined as: A) blood in the anterior chamber of the eye. B) an acute rupture of the globe of the eye. C) inflammation of the iris, cornea, and lens. D) compression of one or both optic nerves.

140. Significant trauma to the face should increase the EMT-B's index of suspicion for a/an: A) airway obstruction. B) displaced mandible. C) basilar skull fracture. D) spinal cord injury.

Page 32

141. You are transporting an immobilized patient with severe facial trauma. As you are preparing to give your radio report to the hospital, the patient begins vomiting large amounts of blood. You should: A) quickly suction his oropharynx. B) turn the backboard on its side. C) reassess his breathing adequacy. D) alert the hospital of the situation.

142. The cricoid cartilage: A) lies superior to the thyroid cartilage in the neck. B) is the only complete circular cartilage of the trachea. C) is easier to see and palpate than the thyroid cartilage. D) lies superior to the cricothyroid membrane in the neck.

143. The small, rounded, fleshy bulge immediately anterior to the ear canal is called the: A) incus. B) pinna. C) tragus. D) stapes.

144. You are dispatched to a convenience store, where the clerk sustained a laceration to his neck during a robbery attempt. During your assessment, you note bright red blood spurting from the laceration. You should: A) apply direct pressure below the lacerated vessel. B) circumferentially wrap a dressing around his neck. C) apply pressure to the closest arterial pressure point. D) apply direct pressure above and below the wound.

145. Elevation of the rib cage during inhalation occurs when: A) the diaphragm descends. B) abdominal contents descend. C) intrathoracic pressure decreases. D) the intercostal muscles contract.

Page 33

146. Pleural fluid is contained between the: A) visceral and parietal pleurae. B) parietal pleura and the heart. C) visceral pleura and the lung. D) parietal pleura and chest wall.

147. Hemoptysis is defined as: A) vomiting blood. B) coughing up blood. C) abnormal blood clotting. D) blood in the pleural space.

148. A flail chest occurs when: A) a segment of the chest wall is detached from the thoracic cage. B) more than three ribs are fractured on the same side of the chest. C) multiple ribs are fractured on both sides of the thoracic cage. D) a segment of fractured ribs bulges during the inhalation phase.

149. Pneumothorax is MOST accurately defined as: A) accumulation of air between the lungs. B) blood collection within the lung tissue. C) accumulation of air in the pleural space. D) blood collection within the pleural space.

150. A rapid, irregular pulse following blunt trauma to the chest is MOST suggestive of a: A) ruptured aorta. B) myocardial contusion. C) pericardial tamponade. D) tension pneumothorax.

Page 34

151. A spontaneous pneumothorax would MOST likely occur as the result of: A) exertion of a person with a congenital lung defect. B) excessive coughing in a patient with pneumonitis. C) abnormally slow breathing in a patient with pleurisy. D) blunt or penetrating trauma to the anterior chest wall.

152. When a hollow organ is punctured during a penetrating injury to the abdomen: A) the abdomen will become instantly distended. B) peritonitis may not develop for several hours. C) it will bleed profusely and rapidly cause shock. D) it commonly protrudes through the injury site.

153. You are transporting a patient with blunt abdominal trauma. The patient is unstable and is experiencing obvious signs and symptoms of shock. Your estimated time of arrival at the hospital is less than 10 minutes. After treating the patient appropriately, you should: A) closely monitor him and reassess him frequently. B) begin documenting the call on the patient care form. C) perform a detailed head-to-toe physical examination. D) forgo the hospital radio report because of his condition.

154. The presence of tachycardia following a significant abdominal injury: A) is always accompanied by hypotension. B) indicates a state of decompensated shock. C) should be assumed to be a sign of shock. D) is most commonly caused by severe pain.

155. All A) B) C) D)

of the following are hollow abdominal organs, EXCEPT for the: liver. bladder. ureters. stomach.

Page 35

156. Which of the following organs is at MOST risk for injury as the result of a pelvic fracture? A) Uterus B) Fallopian tubes C) Urinary bladder D) Liver or spleen

157. A 30-year-old male has a large laceration to his right lower abdominal quadrant with a loop of bowel protruding through the wound. When treating this patient, the EMT-B should recall that the: A) protruding bowel should be kept warm and moist. B) open abdomen rapidly draws heat into the wound. C) wound should be covered with a dry, sterile dressing. D) bowel should be replaced in order to avoid infection.

158. In moving joints, the ends of the bones are covered with: A) articular cartilage. B) synovial tendons. C) muscular fascia. D) gliding cartilage.

159. A/an __________ fracture occurs in the growth section of a child's bone and may lead to bone growth abnormalities. A) greenstick B) diaphyseal C) epiphyseal D) metaphyseal

160. A 54-year-old male accidentally shot himself in the leg while cleaning his gun. Your assessment reveals a small entrance wound to the medial aspect of his right leg. The exit wound is on the opposite side of the leg and is actively bleeding. The patient complains of numbness and tingling in his right foot. You should: A) assess distal pulses as well as sensory and motor functions. B) manually stabilize the leg above and below the site of injury. C) gently manipulate the injured leg until the numbness dissipates. D) control the bleeding and cover the wound with a sterile dressing.
Page 36

161. Of the following musculoskeletal injuries, which is considered to be the LEAST severe? A) Nondisplaced pelvic fracture B) Open fractures of a long bone C) An amputation of an extremity D) Multiple closed long bone fractures

162. When assessing a patient with a possible fracture of the leg, the EMTB should: A) assess proximal circulation. B) compare it to the uninjured leg. C) carefully move it to elicit crepitus. D) ask the patient to move the injured leg.

163. A 22-year-old female was ejected from her car after striking a tree head-on. As you approach her, you note obvious closed deformities to both of her femurs. She is not moving and does not appear to be conscious. You should: A) apply manual stabilization to both of her femurs. B) administer oxygen and perform a rapid assessment. C) assess for a carotid pulse and assist her ventilations. D) stabilize her head and perform an initial assessment.

164. Common signs and symptoms of a sprain include all of the following, EXCEPT: A) swelling. B) deformity. C) ecchymosis. D) point tenderness.

165. Bones are connected to other bones by tough fibrous tissues called: A) bursa. B) tendons. C) cartilage. D) ligaments.

Page 37

166. Smooth muscle is found in all of the following organs, EXCEPT for the: A) esophagus. B) blood vessels. C) myocardium. D) large intestine.

167. A fracture is MOST accurately defined as a: A) total loss of function in a bone. B) break in the continuity of the bone. C) disruption in the midshaft of a bone. D) abnormality in the structure of a bone.

168. During your rapid trauma assessment of a 19-year-old female with multiple trauma, you note bilateral humeral deformities and a deformity to the left midshaft femur. Her skin is diaphoretic and her pulse is rapid and weak. Your partner has appropriately managed the patient's airway and is maintaining manual stabilization of her head. The MOST appropriate treatment for this patient includes: A) applying and inflating the PASG and transporting. B) immobilization to a backboard and rapid transport. C) applying a traction splint to immobilize her femur. D) carefully splinting each of her deformed extremities.

169. The disruption of a joint in which the bone' ends are no longer in contact is called a: A) strain. B) sprain. C) fracture. D) dislocation.

170. Prior to splinting an open extremity fracture, the EMT-B should first: A) assess the injury for crepitation. B) manually stabilize the injury site. C) assess distal neurovascular status. D) cover all bleeding or open wounds.

Page 38

171. An A) B) C) D)

open fracture is MOST accurately defined as a fracture in which: bone ends protrude through the skin. a large laceration overlies the fracture. a bullet shatters the underlying bone. the overlying skin is no longer intact.

172. Skeletal muscle is attached to the bone by tough, ropelike fibrous structures called: A) fascia. B) tendons. C) cartilage. D) ligaments.

173. When splinting a possible fracture of the foot, it is MOST important for the EMT-B to: A) use a pillow as a splint. B) leave the toes exposed. C) apply a pneumatic splint. D) observe for tissue swelling.

174. A temporary loss or alteration of part or all of the brain's abilities to function without physical damage to the brain MOST accurately describes a/an: A) cerebral contusion. B) cerebral concussion. C) intracranial hemorrhage. D) intracerebral hematoma.

175. An A) B) C) D)

epidural hematoma is MOST accurately defined as: bleeding between the skull and dura mater. bleeding between the dura mater and brain. venous lacerations that occur within the brain. an injury caused by a damaged cerebral artery.

Page 39

176. What part of the nervous system controls the body's voluntary activities? A) Central B) Sensory C) Somatic D) Autonomic

177. Distraction injuries of the spine are MOST commonly the result of: A) falls. B) diving. C) hanging. D) compression.

178. When the parasympathetic nervous system is activated: A) blood is shunted away from the digestive organs. B) the heart rate decreases and the blood vessels dilate. C) hormones are released that prepare the body for stress. D) the heart rate increases and the blood pressure increases.

179. Any unconscious trauma patient should be assumed to have: A) a history of diabetes mellitus. B) an accompanying spinal injury. C) a severe intracranial hemorrhage. D) internal bleeding in the abdomen.

180. When immobilizing a trauma patient's spine, the EMT-B manually stabilizing the head should not let go until: A) an appropriately sized cervical collar has been applied. B) the patient has been secured to the ambulance stretcher. C) the head has been stabilized with lateral immobilization. D) the patient has been completely secured to the backboard.

Page 40

181. After your partner assumes manual in-line stabilization of a patient's head, you should: A) apply an appropriately sized rigid cervical collar. B) assess distal neurovascular status in the extremities. C) thoroughly palpate the patient's head for deformities. D) use four people to log roll the patient onto a backboard.

182. Unless he or she is critically ill or injured, you should generally begin your assessment of a toddler: A) in the ambulance. B) at the hands or feet. C) from head to toe. D) en route to the hospital.

183. When assessing an 8-year-old child, you should: A) refrain from taking a blood pressure. B) talk to the child, not just the caregiver. C) use a toe-to-head assessment approach. D) rely solely on the parent for information.

184. Which of the following should make you MOST suspicious for child abuse? A) Worried or crying parents B) A laceration to the child's chin C) Willingness of the child to speak D) Bruises in various stages of healing

185. Common causes of seizures in children include all of the following, EXCEPT: A) infection. B) hyperglycemia. C) electrolyte imbalances. D) poisonings or ingestion.

Page 41

186. The normal respiratory rate for a newborn should not exceed ____ breaths per minute. A) 50 B) 60 C) 70 D) 80

187. When caring for an ill 5-year-old male, you should: A) not expect him to be able to identify areas of pain when asked. B) gain his trust by explaining that a painful procedure will not hurt. C) avoid separating him from his parents or caregivers when possible. D) defer painful procedures, even if the child is clinically unstable.

188. Because a tracheostomy tube bypasses the nose and mouth: A) secretions can build up in and around the tube. B) the risk of a local infection is significantly high. C) bleeding or air leakage may occur around the tube. D) severe swelling of the trachea and bronchi can occur.

189. Immediate transport is indicated for a child when he or she: A) is experiencing mild to moderate pain. B) falls from a height that is greater than 5'. C) has a possible closed fracture of the radius. D) has a history suggestive of a serious illness.

190. All A) B) C) D)

of the following are normal findings in an infant or child, EXCEPT: quiet breathing. fear or anxiety. belly breathing. head bobbing.

Page 42

191. When assessing the heart rate of a 6-month-old infant, you should palpate the brachial or ________ artery. A) radial B) carotid C) femoral D) popliteal

192. When inserting an oropharyngeal airway in an infant or child, you should: A) place padding under the child's head. B) ensure his or her neck is hyperextended. C) insert it until the flange rests on the teeth. D) depress the tongue with a tongue depressor.

193. A 2-year-old female has experienced a seizure. When you arrive at the scene, the child is conscious, crying, and clinging to her mother. Her skin is hot and moist. The mother tells you that the seizure lasted approximately 5 minutes. She further tells you that her daughter has no history of seizures, but has had a recent ear infection. You should: A) allow the mother to drive her daughter to the hospital. B) attempt cooling measures, offer oxygen, and transport. C) place the child in cold water to attempt to reduce her fever. D) suspect that the child has meningitis and transport at once.

194. Which of the following newborns requires positive-pressure ventilations and chest compressions? A) Central cyanosis; respirations, fast; pulse, 100 beats/min B) Central cyanosis; respirations, slow; pulse, 50 beats/min C) Peripheral cyanosis; respirations, fast; pulse, 90 beats/min D) Peripheral cyanosis; respirations, absent; pulse, 80 beats/min

Page 43

195. A 6-year-old male presents with acute respiratory distress. His mother states that she saw him put a small toy into his mouth shortly before the episode began. The child is conscious, obviously frightened, and is coughing forcefully. You should: A) carefully look into his mouth and remove the object if you see it. B) encourage him to cough, give oxygen as tolerated, and transport. C) deliver a series of five back blows and then reassess his condition. D) place the child in a supine position and perform abdominal thrusts.

196. With age, the spine stiffens as a result of shrinkage of the intervertebral disc spaces, and the vertebrae become brittle. This increases the risk of: A) severe kyphosis. B) distracting injuries. C) spinal cord tearing. D) compression fractures.

197. General communication techniques with the elderly include all of the following, EXCEPT: A) being aware of how you present yourself. B) not assuming he or she knows who you are. C) speaking loudly and distinctly to the patient. D) looking directly at the patient when speaking.

198. Poor maintenance of home, poor personal care, and dietary neglect are all possible indications of ____________ elder abuse. A) physical B) financial C) emotional D) psychological

Page 44

199. Common causes of delirium in the elderly patient include all of the following, EXCEPT: A) acute hypovolemia. B) Alzheimer's disease. C) low blood sugar level. D) decreased cerebral perfusion.

200. In contrast to a living will, a do not resuscitate (DNR) order becomes valid when: A) the patient has a terminal illness. B) the patient develops cardiac arrest. C) the patient is in a healthcare setting. D) it is signed by three or more physicians.

201. Which of the following statements regarding communications with the elderly is MOST correct? A) The majority of elderly patients are hearing or visually impaired. B) Attempt to calm the elderly patient by using his or her first name. C) Explain the justification for a procedure after it has been completed. D) Older patients have difficulty understanding when they are stressed.

202. Relatively minor hyperextension injuries to the spine can result in central cord syndrome, a dysfunction characterized by: A) permanent absence of sensory and motor functions to one side of the body. B) absent or weak sensory function more pronounced in the lower extremities. C) temporarily disabled sensory and motor functions in the lower extremities. D) weak or absent motor function more pronounced in the upper extremities.

Page 45

203. A geriatric patient's chief complaint: A) is typically grossly obvious to the EMT-B. B) is usually related to a single disease process. C) usually presents with fairly classic symptoms. D) may be an exacerbation of a chronic problem.

204. Fractures of the pelvis in older patients often occur as the result of a combination of: A) osteoporosis and low-energy trauma. B) increased bone density and car crashes. C) arthritic joints and high-energy trauma. D) acetabular separation and severe falls.

205. Patients who have experienced even minor-appearing head injuries should be suspected of having a brain injury, especially if they: A) have minor abrasions to the head area. B) are taking blood-thinning medications. C) do not have deformities to the skull. D) have a history of Alzheimer's disease.

206. A 69-year-old female was involved in a motor vehicle crash. She is semiconscious with a blood pressure of 80/50 mm Hg and a heart rate of 74 beats/min and weak. Her daughter, who was uninjured in the crash, tells you that her mother has a history of hypertension and takes Inderal. Considering the fact that this patient is probably in shock, what is the MOST likely explanation for the absence of tachycardia? A) Deterioration of the cardiac conduction system B) Intrathoracic bleeding and cardiac compression C) The effects of her antihypertensive medication D) Failure of the parasympathetic nervous system

Page 46

207. You have immobilized the spine of an 81-year-old female with a head injury. During transport, you note that she is experiencing signs of intracranial pressure. In addition to reassessing her ABCs and providing the appropriate treatment, you should: A) elevate the foot end of the backboard 6. B) place several layers of blankets on her. C) elevate the head end of the backboard. D) allow her to sit up on the backboard.

208. When parking your ambulance at the scene of a motor vehicle crash, you should position the ambulance: A) 50' past the scene on the opposite side of the road. B) 100' past the scene on the same side of the road. C) 50' before the scene on the same side of the road. D) alongside the scene to rapidly access the patient(s).

209. When driving an ambulance on a multilane highway in an emergency mode, you should: A) pass other drivers on the right side. B) remain in the extreme left-hand lane. C) remain in the extreme right-hand lane. D) drive in the center lane of the highway.

210. It is 10:30 p.m. and you have requested air medical transport for a critically injured patient. As you arrive at the designated landing zone, you should: A) mark the proposed landing area with road flares. B) survey the area for power lines or other hazards. C) provide the flight crew with a patient status update. D) mark the landing site with personnel with flashlights.

Page 47

211. When working at the scene of a motor vehicle crash at night, you should NOT use: A) road flares. B) reflective vests. C) portable floodlights. D) intermittent flashing devices.

212. When determining the exact location and position of the patient(s) in a wrecked vehicle, you and your team should consider all of the following, EXCEPT: A) the position of the crashed vehicle. B) hazards that pose a risk to rescuers. C) the make and model of the vehicle. D) the possibility of vehicle instability.

213. The scene size-up at a car crash or other incident: A) is a continual process until the incident is terminated. B) should be performed by the most experienced EMT-B. C) is a quick visual assessment of the scene prior to entry. D) determines who is allowed to safely enter the hot zone.

214. Upon arriving at the scene of a law enforcement tactical situation, you should ensure your own safety and then: A) begin immediate triage of any injured personnel. B) report to the incident commander for instructions. C) locate all injured personnel and begin treatment. D) apprise medical control of the tactical situation.

215. An EMT-B should NOT attempt to gain access to the patient(s) in a damaged vehicle until: A) the fire department arrives at the scene. B) he or she is sure that the vehicle is stable. C) all occupants in the vehicle are accounted for. D) heavy-duty extrication equipment is available.

Page 48

216. You have a critically injured patient in the back of your ambulance, ready to be transported. There are other injured patients at the scene and it will be approximately 10 minutes before other ambulances will arrive. Law enforcement personnel are at the scene. You should: A) transport the critically injured patient to a trauma center. B) direct a police officer to monitor the patients as you transport. C) remain at the scene until at least one other ambulance arrives. D) assign the least injured patient the task of caring for the others.

217. A mass casualty incident is MOST accurately defined as: A) an incident that involves more than five critically injured or ill patients. B) an incident where patients have been exposed to hazardous materials. C) an incident in which at least half of the patients are critically injured. D) an incident that greatly taxes or depletes a system's available resources.

218. If the safety officer issues a command or directive to the EMT-B, the EMT-B should: A) immediately carry out the order or directive. B) disregard the order and follow local protocols. C) notify medical control for proper authorization. D) confirm the order with the incident commander.

219. Injuries or conditions that would be classified as first priority (red tag) include all of the following, EXCEPT: A) severe medical problems. B) fractures of multiple long bones. C) any airway or breathing difficulty. D) uncontrolled or severe hemorrhage.

Page 49

220. Unlike a mass-casualty incident, a natural disaster: A) often requires personnel to remain on scene for several days. B) exists when there are more than 100 critically injured patients. C) is typically short-lived and does not require as much manpower. D) usually does not require the incident command system process.

221. ______ rays easily penetrate through the human body and require several inches of lead or concrete to prevent penetration. A) Beta B) Alpha C) Neutron D) Gamma

222. A weapon of mass destruction (WMD) is MOST accurately defined as: A) a device or agent used to destroy a specific area or region within a given geographic location. B) any agent used to bring about mass death, casualties, or massive infrastructural damage. C) a nuclear or chemical weapon that can be launched from one country to another country. D) any device used for the express purpose of creating carnage in an effort to make a particular point.

223. You are dispatched to the scene of a building explosion. Upon arrival, you see people frantically fleeing the building, screaming, Everyone is passing out! You should: A) carefully assess the situation and ensure your own safety. B) assist with the evacuation and begin triaging the patients. C) notify dispatch and state that a terrorist attack has occurred. D) contact the FBI immediately and report the current situation.

224. The skin lesions associated with smallpox: A) initially form on the lower trunk. B) are of different shapes and sizes. C) develop early during the disease. D) are identical in their development.
Page 50

Answer Key - FHS EMT review exam1 1. 2. 3. 4. 5. 6. 7. 8. 9. 10. 11. 12. 13. 14. 15. 16. 17. 18. 19. 20. 21. 22. 23. 24. 25. 26. 27. 28. 29. 30. 31. 32. 33. 34. 35. 36. 37. 38. A C B D B B B D B B B A D A A B B D C B A D D D B A B D C A C D C C B D B A
Page 51

39. 40. 41. 42. 43. 44. 45. 46. 47. 48. 49. 50. 51. 52. 53. 54. 55. 56. 57. 58. 59. 60. 61. 62. 63. 64. 65. 66. 67. 68. 69. 70. 71. 72. 73. 74. 75. 76. 77. 78.

C D A D D B A C B C A B B C B C D A A A A A C C B A C B C C C A B C D B B B A A
Page 52

79. 80. 81. 82. 83. 84. 85. 86. 87. 88. 89. 90. 91. 92. 93. 94. 95. 96. 97. 98. 99. 100. 101. 102. 103. 104. 105. 106. 107. 108. 109. 110. 111. 112. 113. 114. 115. 116. 117. 118.

B D B B D B C D B B D C D C A A A D A A D C B A A D B D C B A C D A A D B C D D
Page 53

119. 120. 121. 122. 123. 124. 125. 126. 127. 128. 129. 130. 131. 132. 133. 134. 135. 136. 137. 138. 139. 140. 141. 142. 143. 144. 145. 146. 147. 148. 149. 150. 151. 152. 153. 154. 155. 156. 157. 158.

B A D B D A C A A A A C D D D A C B B C A D B B C D D A B A C B A B A C A C A A
Page 54

159. 160. 161. 162. 163. 164. 165. 166. 167. 168. 169. 170. 171. 172. 173. 174. 175. 176. 177. 178. 179. 180. 181. 182. 183. 184. 185. 186. 187. 188. 189. 190. 191. 192. 193. 194. 195. 196. 197. 198.

C D A B D B D C B B D D D B B B A C C B B D B B B D B B C A D D C D B B B D C A
Page 55

199. 200. 201. 202. 203. 204. 205. 206. 207. 208. 209. 210. 211. 212. 213. 214. 215. 216. 217. 218. 219. 220. 221. 222. 223. 224.

B B D D D A B C C B B B A C A B B C D A B A D B A D

Page 56

Name: ___________________________________ Date: ______________

1. What is the correct ratio of ventilations to compressions when performing two-rescuer child CPR? A) 1:3 B) 1:5 C) 1:10 D) 2:15

2. Which of the following is NOT a BLS intervention? A) Abdominal thrusts B) Chest compressions C) Cardiac monitoring D) Automated defibrillation

3. When performing CPR on a child, you should compress the chest: A) until a radial pulse is felt. B) with one or two hands. C) to a depth of 1 to 2 inches. D) 70 to 80 times per minute.

4. In MOST cases, cardiopulmonary arrest in infants and children is caused by: A) a drug overdose. B) respiratory arrest. C) severe chest trauma. D) a cardiac dysrhythmia.

5. Which of the following techniques should you use to dislodge a foreign body airway obstruction in a patient who is in an advanced stage of pregnancy or who is very obese? A) Back blows B) Finger sweeps C) Chest thrusts D) Abdominal thrusts

Page 1

6. The MOST important consideration at the scene of a hazardous material incident is: A) identifying the material. B) calling the HazMat team. C) evacuation of bystanders. D) ensuring personal safety.

7. The MOST effective way to preserve your own body heat when functioning in cold, wet weather is to: A) wear a heavy, thick jacket or coat. B) avoid outer clothing with zippers. C) wear at least three layers of clothing. D) wear socks made of heavy-duty cotton.

8. Which of the following is the MOST effective strategy for managing stress? A) Frequently reflect on troublesome calls. B) Avoid friends and interests outside of EMS. C) Request overtime to increase your income. D) Focus on delivering high-quality patient care.

9. Which of the following signs would indicate that your partner is experiencing significant stress? A) Increased interest in daily activities B) Isolation from the rest of the crew C) Hyperactivity during an EMS call D) Acute increase in his or her appetite

10. What type of stress reaction occurs when an EMT-B is exposed to many insignificant stressors over a period of several months or years? A) Acute stress reaction B) Cumulative stress reaction C) Posttraumatic stress reaction D) Critical incident stress reaction

11. Which of the following situations requires you to notify the appropriate authorities? A) Cardiac arrest B) Drug overdose C) Attempted suicide D) Accidental knife wound
Page 2

12. Which of the following statements regarding the Good Samaritan Law is correct? A) It provides the EMT-B immunity from a lawsuit. B) It guarantees that you will not be held liable if sued. C) The law does not protect EMT-Bs who are off duty. D) It will not protect you in cases of gross negligence.

13. In many states, a minor may be treated as an adult for the purpose of consenting to or refusing medical treatment if the minor: A) possesses a valid driver's license. B) is mentally competent and able to refuse. C) has a poor relationship with his or her parents. D) is married and lives independent of his or her parents.

14. The EMT-B's scope of practice within his or her local response area is defined by the: A) medical director. B) state EMS office. C) EMS supervisor. D) local health district.

15. Which of the following components are needed to prove negligence? A) Abandonment, breach of duty, damages, and causation B) Duty to act, abandonment, breach of duty, and causation C) Duty to act, breach of duty, injury/damages, and causation D) Breach of duty, injury/damages, abandonment, and causation

16. Which of the following chemicals is released by the adrenal glands and increases the heart rate and blood pressure? A) Insulin B) Estrogen C) Thyroxin D) Epinephrine

17. What three bones make up the shoulder girdle? A) Clavicle, scapula, humerus B) Acromion, clavicle, scapula C) Acromion, scapula, humerus D) Acromion, humerus, clavicle

Page 3

18. The topographic term used to describe the part of the body that is nearer to the feet is: A) dorsal. B) inferior. C) internal. D) superior.

19. In relation to the wrist, the elbow is: A) distal. B) medial. C) lateral. D) proximal.

20. The major artery that supplies all other arteries with blood is the: A) aorta. B) carotid. C) brachial. D) femoral.

21. Which of the following medical history questions would be of LEAST pertinence in an acute situation? A) Does the pain stay in your chest? B) Does your mother have diabetes? C) Has this ever happened to you before? D) Are there medications that you can't take?

22. When taking a patient's blood pressure in an upper extremity, the diaphragm of the stethoscope is normally placed over which artery? A) Brachial B) Femoral C) Popliteal D) Radial

23. With regard to assessment of a patient's cardiovascular status, capillary refill time (CRT) is MOST reliable in: A) children who are younger than 6 years of age. B) patients who are significantly hypotensive. C) children who are older than 6 years of age. D) patients with decreased peripheral perfusion.
Page 4

24. Which of the following MOST accurately describes the term chief complaint? A) The information about the call provided to you by the dispatcher B) The major signs or symptoms that the patient reports when asked C) The general appearance of the patient upon your initial encounter D) The symptoms that the patient denies having when he or she is asked

25. Which of the following is the MOST accurate guide to palpating a pulse? A) Avoid compressing the artery against a bone or solid structure. B) Place the tips of your index and long fingers over the pulse point. C) Use your thumb to increase the surface area that you are palpating. D) Apply firm pressure to the artery with your ring and little fingers.

26. When the shoulder girdle is aligned over the pelvis during lifting: A) the weight is exerted straight down the vertebrae. B) the hands can be held further apart from the body. C) the muscles of the back experience increased strain. D) the risk of back injuries is significantly increased.

27. A folding or portable stretcher is MOST beneficial when: A) a second patient must be transported on the squad bench of the ambulance. B) an injured patient cannot be placed on a long board due to severe back pain. C) a conscious, alert patient must be carried down several flights of steep stairs. D) a patient requires full spinal immobilization when spinal injury is suspected.

Page 5

28. You have two patients that were involved in a motor-vehicle crash, when their SUV struck a treeone with neck and back pain, and the other with a deformed left femur. The patient with the deformed femur states that he does not want to be placed on a hard board, nor does he want a collar around his neck. What is the MOST appropriate and practical method of securing these patients and placing them into the ambulance? A) Immobilize both patients with a cervical collar and long backboard based on the mechanism of injury; place one on the wheeled stretcher and the other on the squad bench. B) Immobilize the patient with neck and back pain on a long backboard and place him on the wheeled stretcher; place the patient with the deformed femur on a folding stretcher secured to the squad bench. C) Immobilize the patient with neck and back pain on a long backboard and place him on the squad bench; allow the patient with the deformed femur to sit on the wheeled stretcher. D) Apply a traction splint to the patient with the deformed femur and place him on the wheeled stretcher; place the patient with neck and back pain on the squad bench immobilized with a cervical collar and scoop stretcher.

29. The proper technique for using the power grip is to: A) lift with your palms up. B) rotate your palms down. C) hold the handle with your fingers. D) position your hands about 6" apart.

30. Which of the following is the MOST appropriate device to use when immobilizing a patient with a suspected spinal injury? A) Long backboard B) Scoop stretcher C) Portable stretcher D) Wheeled stretcher

31. In what manner should you act and speak with a patient? A) Passive B) Authoritative C) Loud and official D) Calm and confident

Page 6

32. Two-way communication that requires the EMT-B to "push to talk" and "release to listen" describes what mode of communication? A) Duplex B) Simplex C) Multiplex D) Mediplex

33. A patient's refusal for EMS treatment and/or transport must be: A) an informed refusal. B) ignored by the EMT. C) reported to the police. D) witnessed by a notary.

34. Which of the following statements regarding a patient refusal is MOST correct? A) A patient who consumed a few beers will likely be able to refuse EMS treatment. B) Advice given to a patient who refuses EMS treatment should not be documented. C) A mentally competent adult has the legal right to refuse EMS care and transport. D) Documentation of proposed care is unnecessary if the patient refuses treatment.

35. When you begin a verbal report, you should state the patient's age, sex, and: A) chief complaint. B) any known allergies. C) past medical history. D) emergency care given.

Page 7

Answer Key - Practice Exam 1 1. 2. 3. 4. 5. 6. 7. 8. 9. 10. 11. 12. 13. 14. 15. 16. 17. 18. 19. 20. 21. 22. 23. 24. 25. 26. 27. 28. 29. 30. 31. 32. 33. 34. 35. D C B B C D C D B B C D D A C D A B D A B A A B B A A B A A D B A C A

Page 8

Name: __________________________ Date: _____________

1. Cyanosis of the skin is caused by: A) increased blood oxygen. B) peripheral vasodilation. C) venous vasoconstriction. D) decreased blood oxygen.

2. A patient with spontaneous respirations is breathing: A) at a normal rate. B) with shallow depth. C) without difficulty. D) without assistance.

3. An adult patient who is NOT experiencing difficulty breathing will: A) be able to speak in complete sentences without unusual pauses. B) assume a position that will facilitate effective and easy breathing. C) exhibit an indentation above the clavicles and in between the ribs. D) have a respiratory rate that is between 20 and 24 breaths/min.

4. Typical methods of assessing a patient's breathing include all of the following, EXCEPT: A) listening to breath sounds with a stethoscope. B) observing for nasal flaring during inhalation. C) observing the chest for adequate rise and fall. D) quickly scanning the chest for obvious movement.

5. During the primary assessment, circulation is evaluated by assessing: A) skin quality, blood pressure, and capillary refill. B) pulse quality, external bleeding, and skin condition. C) blood pressure, pulse rate, and external bleeding. D) external bleeding, skin condition, and capillary refill.

6. External bleeding from an extremity can usually be controlled by a combination of: A) direct pressure and elevation. B) direct pressure and a tourniquet. C) elevation and chemical ice packs. D) elevation and pressure point control.

Page 1

7. When palpating a patient's pulse, you note that there is a short interval between pulsations. This indicates that the pulse is: A) slow. B) rapid. C) irregular. D) thready.

8. Pain that moves from its point of origin to another body location is said to be: A) radiating. B) referred. C) palliating. D) provoking.

9. When auscultating the blood pressure in a patient's upper extremity, you should place the diaphragm (head) of the stethoscope over the _________ artery. A) radial B) apical C) femoral D) brachial

10. A decrease in the blood pressure may indicate: A) a loss of vascular tone. B) arterial constriction. C) increased blood volume. D) forceful cardiac contraction.

11. An injured patient is assigned a total score of 9 on the GCS. He is assigned a score of 2 for eye opening, a score of 3 for verbal response, and a score of 4 for motor response. Which of the following clinical findings is consistent with his GCS score? A) opens eyes in response to voice, makes incomprehensible sounds, localizes pain B) opens eyes in response to pain, uses inappropriate words, withdraws from pain C) opens eyes spontaneously, is confused when spoken to, exhibits abnormal flexion D) eyes remain closed, makes incomprehensible sounds, exhibits abnormal extension

12. The diastolic pressure represents the: A) average pressure against the arterial walls during a cardiac cycle. B) minimum amount of pressure that is always present in the arteries. C) increased arterial pressure that occurs during ventricular contraction. D) difference in pressure between ventricular contraction and relaxation.

Page 2

13. Which of the following abnormal breath sounds indicates obstruction of the upper airway? A) rales B) stridor C) crackles D) rhonchi

14. You respond to a call for a female pedestrian who has been struck by a car. As your partner maintains manual stabilization of her head, you perform a primary assessment. She is unconscious, has ineffective breathing, and has bloody secretions in her mouth. You should: A) assist her breathing with a bag-mask device. B) quickly insert an oropharyngeal airway. C) assess the rate and quality of her pulse. D) immediately suction her oropharynx.

15. What part of the patient assessment process focuses on obtaining additional information about the patient's chief complaint and any medical problems he or she may have? A) history taking B) general impression C) primary assessment D) secondary assessment

16. When is it MOST appropriate to consider requesting additional ambulances at an accident scene? A) after you have triaged all the critical patients B) when you determine there are multiple patients C) after noncritical patients have been identified D) when all the deceased patients are accounted for

17. Supplemental oxygen via nonrebreathing mask should be administered to patients: A) who are semiconscious with shallow respirations. B) with rapid respirations and a reduced tidal volume. C) who have accessory muscle use and slow breathing. D) with difficulty breathing and adequate tidal volume.

Page 3

18. A palpable pulse is created by: A) the pressure of circulating blood against the walls of the arteries. B) the pressure that is caused when venous blood returns to the heart. C) pressure waves through the arteries caused by cardiac contraction. D) electrical conduction in the heart producing ventricular contraction.

19. The chief complaint is MOST accurately defined as the: A) most life-threatening condition that you discover. B) condition that exacerbates an underlying problem. C) most serious thing the patient is concerned about. D) gross physical signs that you detect on assessment.

20. Which of the following is the MOST effective method of assessing the quality of air movement in the lungs? A) evaluating the patient's chest for cyanosis B) applying a pulse oximeter and monitoring the SpO2 C) auscultating breath sounds with a stethoscope D) looking for the presence of accessory muscle use

21. When performing a full-body scan on a supine patient, what part of the body is typically assessed last? A) abdomen B) posterior C) extremities D) anterior chest

22. Which of the following statements regarding the blood pressure is correct? A) The systolic pressure represents ventricular relaxation. B) Blood pressure falls early in patients with hypoperfusion. C) Blood pressure is the most reliable indicator of perfusion. D) Blood pressure is usually not measured in children younger than 3 years of age.

23. Upon arriving at a potentially unsafe scene, you should: A) remove all bystanders. B) request another ambulance. C) move the patient to safety. D) ensure that you are safe.

Page 4

24. In responsive patients that are older than 1 year of age, you should palpate the pulse at the ________ artery. A) radial B) carotid C) brachial D) femoral

25. In patients with deeply pigmented skin, changes in color may be apparent only in certain areas, such as the: A) back of the neck. B) forehead and face. C) dorsum of the hand. D) lips or oral mucosa.

26. The goal of the full-body scan that is performed during the secondary assessment is to: A) detect and treat all nonlife-threatening injuries. B) assess only the parts of the body that are injured. C) definitively rule out significant internal injuries. D) locate injuries not found in the primary assessment.

27. You are assessing a 72-year-old man with abdominal pain. The patient is sitting in a chair; he is conscious, alert, and calm. As you are talking to the patient, your partner discreetly directs your attention to a handgun, which is located on a nearby table. You should: A) immediately cease all patient care, carefully back out of the residence, and request law enforcement assistance. B) direct your partner to move the gun to a safe area and then advise the patient that his weapon has been secured. C) document the presence of the weapon, including its specific location, and continue your assessment of the patient. D) position yourself in between the patient and the gun and ask your partner to request law enforcement assistance.

28. You receive a call to a local daycare center for an unresponsive 8-month-old infant. Upon arrival, you perform an assessment and determine that the infant is not breathing. Your next action should be to: A) open the airway and give 2 rescue breaths. B) begin chest compressions and request backup. C) immediately transport the child to the hospital. D) assess for a brachial pulse for 5 to 10 seconds.

Page 5

29. A patient with profuse sweating is referred to as being: A) flushed. B) plethoric. C) diaphoretic. D) edematous.

30. Poor peripheral circulation will cause the skin to appear: A) pink. B) ashen. C) flushed. D) cyanotic.

31. When performing a reassessment of your patient, you should first: A) obtain updated vital signs. B) reassess your interventions. C) repeat the primary assessment. D) confirm medical history findings.

32. A 39-year-old male sustained a stab wound to the groin during an altercation at a bar. As you approach the patient, you note that he is conscious, is screaming in pain, and is attempting to control the bleeding, which is bright red and spurting from his groin area. You should: A) ensure that his airway is patent. B) apply direct pressure to the wound. C) elevate his legs and keep him warm. D) administer 100% supplemental oxygen.

33. When interviewing a patient, you can show him or her that you understand the situation by: A) repeating statements back to him or her. B) using medical terminology whenever possible. C) maintaining constant eye contact with him or her. D) interrupting him or her as needed for clarification.

34. You respond to the residence of a 62-year-old male who is unresponsive. Your primary assessment reveals that he is apneic and pulseless. You should: A) start CPR and attach the AED as soon as possible. B) ask the family if the patient has a terminal disease. C) perform CPR and transport the patient immediately. D) notify dispatch and request a paramedic ambulance.

Page 6

35. An elderly patient has fallen and hit her head. You assess her level of consciousness as unresponsive using the AVPU scale. Your initial care should focus on: A) obtaining baseline vital signs. B) gathering medical history data. C) providing immediate transport. D) airway, breathing, and circulation.

36. The normal respiratory rate for an adult should range from: A) 10 to 15 breaths per minute. B) 12 to 20 breaths per minute. C) 18 to 24 breaths per minute. D) 24 to 28 breaths per minute.

37. When assessing a patient's abdomen, you will typically evaluate for all of the following, EXCEPT: A) subcutaneous emphysema. B) open wounds or eviscerations. C) gross bleeding and tenderness. D) rigidity and obvious bleeding.

38. When approaching a 32-year-old male who is complaining of traumatic neck pain, you should: A) ensure that the patient can see you approaching him. B) approach him from behind and ask him not to move. C) stand behind him and immediately stabilize his head. D) assess his mental status by having him move his head.

39. Upon arriving at the scene of a patient with difficulty breathing, you determine that the scene is safe. You enter the residence and find the patient sitting in a chair in obvious distress. Your first action should be to: A) ask the patient what's wrong. B) obtain a set of baseline vital signs. C) assess the patient's airway status. D) introduce yourself to the patient.

Page 7

40. You respond to the scene of a motor vehicle collision. Upon arrival, you find the driver, a young female, sitting on the curb. She is confused, is in obvious respiratory distress, and has pale, moist skin. As your partner manually stabilizes her head, you perform a primary assessment. After performing any immediate livesaving treatment, you should: A) perform a rapid scan of her entire body and prepare for immediate transport. B) assess her vital signs, secure her to a backboard, and transport her immediately. C) fully immobilize her spine, load her into the ambulance, and assess her vital signs. D) identify the specific areas of her injuries and focus your assessment on those areas.

41. When using the pulse oximeter as part of your assessment of a patient, it is important to remember that: A) pulse oximetry is especially useful in patients who have cold extremities because vasoconstriction forces blood to the capillary beds. B) carbon monoxide has no effect on pulse oximetry readings because the pulse oximeter presumes that oxygen is saturating the hemoglobin. C) as long as the patient's oxygen saturation is greater than 95%, oxygen is usually not necessary, even if the patient has respiratory distress. D) any situation that causes vasoconstriction or loss of red blood cells, such as anemia or bleeding, may result in an inaccurate or misleading value.

42. Which of the following situations or conditions warrants immediate transport? A) mild pain in the lower abdomen B) severe chest pain and cool, pale skin C) decreased ability to move an extremity D) responsiveness and ability to follow commands

43. After performing a primary assessment, a rapid scan of the body should be performed in order to: A) determine the need for spinal motion restriction precautions. B) identify less obvious injuries that require immediate treatment. C) look specifically for signs and symptoms of inadequate perfusion. D) find and treat injuries or conditions that do not pose a threat to life.

44. In infants and small children, skin color should be assessed on the: A) forehead. B) palms and soles. C) chest and abdomen. D) underside of the arms.

Page 8

45. A pulse with a consistent pattern is considered to be: A) weak. B) strong. C) regular. D) irregular. 46. Which of the following questions would you ask a patient to ascertain the M in the SAMPLE history? A) Have you ever had any major surgeries? B) How long have you had your chest pain? C) How much Tylenol do you take each day? D) When was the last time you ate a meal?

47. Which of the following questions is used to determine a patient's chief complaint? A) What seems to be the matter? B) When did the chest pain begin? C) Are you having trouble breathing? D) Do you have a history of diabetes?

48. A patient with high blood pressure would be expected to have skin that is: A) flushed and red. B) mottled and cool. C) pale and moist. D) cyanotic and dry.

49. In which of the following situations is a pertinent negative identified? A) A 50-year-old woman states that nothing makes her chest pain better or worse. B) A 53-year-old man with dizziness also tells you that he has vomited three times. C) A 56-year-old woman states that her chest hurts every time she takes a deep breath. D) A 59-year-old man complains of crushing chest pain but denies shortness of breath.

50. A 29-year-old male with a head injury opens his eyes when you speak to him, is confused as to the time and date, and is able to move all of his extremities on command. His Glasgow Coma Scale (GCS) score is: A) 10. B) 12. C) 13. D) 14.

Page 9

51. You should gently palpate a patient's pelvis only if: A) you note gross deformity to the pelvic area. B) the patient does not complain of pelvic pain. C) the MOI suggests pelvic trauma. D) the possibility of a pelvic fracture has been ruled out.

52. A patient's short-term memory is MOST likely intact if he or she correctly answers questions regarding: A) time and place. B) date and event. C) event and person. D) person and place.

53. When assessing motor function in a conscious patient's lower extremities, you should expect the patient to: A) wiggle his or her toes on command. B) feel you touching the extremity. C) note any changes in temperature. D) identify different types of stimuli.

54. Which of the following conditions would MOST likely cause the pupils to remain significantly constricted? A) severe cerebral hypoxia B) intracranial hemorrhaging C) overdose of an opiate drug D) oculomotor nerve pressure

55. Which of the following conditions would be LEAST likely to cause an altered level of consciousness? A) drug overdose B) inadequate perfusion C) acute anxiety D) poisoning

56. After the first 60 minutes of experiencing a significant injury: A) the patient's blood pressure elevates significantly. B) the patient's injuries will most likely be irreparable. C) most patients will die secondary to internal bleeding. D) the body's ability to compensate for shock decreases.

Page 10

57. Which of the following patient responses would establish the E in the SAMPLE history? A) I was in the hospital a week ago. B) I am not having any difficulty breathing. C) The chest pain started about 45 minutes ago. D) I was mowing the lawn when the pain began.

58. A full-body scan should be performed on: A) stable patients who are able to tell you exactly what happened. B) all patients with traumatic injuries who will require EMS transport. C) responsive medical patients and patients without a significant MOI. D) patients with a significant MOI and unresponsive medical patients.

59. If you cannot palpate a pulse in an unresponsive patient, you should: A) attach an automated external defibrillator (AED) at once. B) immediately begin cardiopulmonary resuscitation (CPR). C) palpate at another pulse site. D) assess for adequate breathing.

60. The goal of the primary assessment is to: A) determine if the patient's problem is medical or trauma. B) identify patients that require transport to a trauma center. C) determine the need to perform a head-to-toe assessment. D) identify and rapidly treat all life-threatening conditions.

61. The pressure exerted against the walls of the artery when the left ventricle contracts is called the: A) blood pressure. B) systolic pressure. C) diastolic pressure. D) pulse pressure.

62. When a patient's respirations are shallow: A) chest rise will be easily noticeable. B) tidal volume is markedly reduced. C) oxygenation occurs more efficiently. D) carbon dioxide elimination is increased.

Page 11

63. The Golden Period begins when an injury occurs and ends when: A) the patient receives definitive care. B) the patient is admitted to the ICU. C) you depart the scene for the hospital. D) you arrive at the emergency department.

64. Which of the following scenarios does NOT involve the presence of any symptoms? A) a 44-year-old male with abdominal pain and severe dizziness B) a 49-year-old female with blurred vision and ringing in the ears C) a 55-year-old male with a severe headache and 2 days of nausea D) a 61-year-old female who is unconscious with facial cyanosis

65. A crackling sound produced by air bubbles under the skin is called: A) crepitus B) rhonchi. C) Korotkoff sounds. D) subcutaneous emphysema.

66. The full-body scan of a patient that occurs following the primary assessment should take no longer than: A) 30 seconds. B) 60 to 90 seconds. C) 90 to 120 seconds. D) 120 to 180 seconds.

67. A 50-year-old male presents with an altered mental status. His wife tells you that he had a small stroke 3 years ago but has otherwise been in good health. The patient is responsive but unable to follow commands. After administering oxygen, you should: A) repeat the primary assessment. B) inquire about his family history. C) prepare for immediate transport. D) perform a head-to-toe assessment.

68. Which of the following MOST accurately describes paradoxical movement of the chest wall? A) multiple rib fractures that cause a marked deformity of the chest wall B) a marked decrease in chest wall movement due to abdominal breathing C) only one section of the chest rises on inspiration while another area falls D) one side of the chest wall moves opposite the direction of the other

Page 12

69. When you shine a light into one pupil, the normal reaction of the other pupil should be to: A) dilate. B) not react. C) become larger. D) become smaller.

70. Observations made when forming a general impression of a patient would include all of the following, EXCEPT: A) appearance. B) pulse strength. C) race and gender. D) level of distress.

71. Treatment and transport priorities at the scene of a mass-casualty incident should be determined after: A) a physician arrives at the scene. B) the number of patients is known. C) all the patients have been triaged. D) area hospitals have been notified.

72. During a 30-minute transport of a stable patient, you should reassess him or her at least ________ times. A) 2 B) 3 C) 4 D) 6

73. Palpating the carotid pulse is not recommended in infants because: A) the pulse rate is usually too fast to count. B) a decrease in cerebral perfusion may occur. C) the carotid artery is more anterior in infants. D) you may inadvertently compress the trachea.

74. Which of the following medical history questions would be of LEAST pertinence in an acute situation? A) Does the pain stay in your chest? B) Does your mother have diabetes? C) Has this ever happened to you before? D) Are there medications that you cannot take?

Page 13

75. A 40-year-old male presents with pain to the right upper quadrant of his abdomen. He is conscious and alert with stable vital signs. During your assessment, you note that his skin and sclera are jaundiced. You should suspect: A) acute pancreatitis. B) liver dysfunction. C) gallbladder disease. D) renal insufficiency.

76. A 40-year-old male crashed his motorcycle into a tree. He is semiconscious, has snoring respirations, and has a laceration to the forearm with minimal bleeding. You should: A) apply a cervical collar and suction his airway. B) open his airway with the jaw-thrust maneuver. C) apply a pressure dressing to the patient's arm. D) tilt the patient's head back and lift up on his chin.

77. A 71-year-old female slipped on a rug and fell. She is conscious and alert and complains of severe pelvic pain. Her respirations are 22 breaths/min with adequate depth and her heart rate is 120 beats/min. Which of the following would NOT be appropriate for this patient? A) performing a full-body scan B) 100% supplemental oxygen C) treating her for possible shock D) gentle palpation of the pelvis

78. You should suspect that a patient is experiencing respiratory failure if he or she: A) is restless and is working hard to breathe. B) has an increased heart rate and retractions. C) has bradycardia and diminished muscle tone. D) is anxious, tachycardic, and leaning forward.

79. A blood pressure cuff that is too small for a patient's arm will give a: A) falsely low systolic and diastolic reading. B) falsely high systolic but low diastolic reading. C) falsely high systolic and diastolic reading. D) falsely low systolic but high diastolic reading.

Page 14

80. When palpating the carotid pulse of a responsive older patient, you should: A) avoid compressing both carotid arteries simultaneously. B) ensure that his or her head is in a hyperextended position. C) avoid gentle pressure so that weak pulses can be detected. D) firmly compress the artery because the pulse is often weak.

81. The pulse oximeter is an assessment tool used to evaluate the: A) percentage of red blood cells. B) effectiveness of oxygenation. C) saturation level of venous blood. D) amount of exhaled carbon dioxide.

82. Which of the following statements regarding the mechanism of injury (MOI) is correct? A) A nonsignificant MOI rules out the possibility of serious trauma. B) The MOI may allow you to predict the severity of a patient's injuries. C) The exact location of a patient's injuries can be determined by the MOI. D) A significant MOI always results in patient death or permanent disability.

83. Which of the following is the MOST accurate guide to palpating a pulse? A) Avoid compressing the artery against a bone or solid structure. B) Place the tips of your index and long fingers over the pulse point. C) Use your thumb to increase the surface area that you are palpating. D) Apply firm pressure to the artery with your ring and little fingers.

84. The MOST effective way to determine whether your patient's problem is medical or traumatic in origin is to: A) perform a careful and thorough assessment. B) establish the patient's medical history early. C) take note of the patient's general appearance. D) ask if bystanders are familiar with the patient.

85. An unstable patient should be reassessed at least every: A) 5 minutes. B) 10 minutes. C) 15 minutes. D) 20 minutes.

Page 15

86. Which of the following patients does NOT have an altered mental status? A) a patient with an acute allergic reaction and dizziness B) a diabetic who opens his eyes when you ask questions C) a patient with a head injury who is slow to answer questions D) a patient who overdosed and moans when he is touched

87. While evaluating a patient with chest pain, your partner tells you that the patient's blood pressure is 140/94 mm Hg. The lower number represents the pressure from the: A) atria relaxing. B) atria contracting. C) ventricles relaxing. D) ventricles contracting.

88. Which of the following statements regarding stridor is MOST correct? A) It is a whistling sound heard in the lower airway. B) It is caused by incorrect airway positioning. C) It is a high-pitched, crowing upper airway sound. D) It suggests the presence of fluid in the lungs.

89. Reassessment is performed to determine all of the following, EXCEPT: A) the reason why the patient called EMS. B) the patient's response to your treatment. C) whether or not the patient is deteriorating. D) the nature of any newly identified problems.

90. When assessing the skin of an unconscious patient, you note that it has a bluish tint to it. This finding is called: A) pallor. B) flushing. C) cyanosis. D) mottling.

91. During an EMS call, you should take standard precautions: A) any time before you load the patient into the ambulance. B) after it has been determined that the patient is bleeding. C) immediately after completion of your primary assessment. D) upon exiting the ambulance, but before actual patient contact.

Page 16

92. When evaluating a patient with multiple complaints, the EMT's responsibility is to: A) direct his or her attention to the most obvious signs and symptoms. B) determine which complaint poses the greatest threat to the patient's life. C) definitively rule out serious causes of each of the patient's complaints. D) assess each complaint based on the patient's perception of its seriousness.

93. A low ETCO2 reading, as measured by capnography, would MOST likely be observed if: A) a patient in cardiac arrest is receiving high-quality CPR. B) an endotracheal (ET) tube is correctly placed in the trachea. C) there is an absence or decrease in the level of CO2 in the lungs. D) the cells produce large amounts of CO2 and return it to the lungs. 94. You are dispatched to the county jail for an inmate who is sick. When you arrive, you find the patient, a 33-year-old male, unresponsive. His airway is patent and his respirations are rapid and shallow. Your initial action should be to: A) apply a pulse oximeter. B) request a paramedic unit. C) provide assisted ventilation. D) assess his blood pressure.

95. When palpating a patient's pulse, you note that it is grossly irregular. You should: A) count the pulse rate for at least 30 seconds to ensure accuracy. B) count the number of pulsations in 15 seconds and multiply by four. C) count the number of pulsations in 30 seconds and multiply by two. D) count the pulse rate for a full minute to obtain an accurate reading.

96. Which of the following is an example of a symptom? A) cyanosis B) headache C) tachycardia D) hypertension

97. Normal skin color, temperature, and condition should be: A) pink, warm, and dry. B) pale, cool, and moist. C) pink, warm, and moist. D) flushed, cool, and dry.

Page 17

98. A properly sized blood pressure cuff should cover: A) two thirds the length from the armpit to the crease in the elbow. B) one half the length between the armpit to the crease in the elbow. C) one third the length from the armpit to the crease at the elbow. D) the entire upper arm between the armpit and the crease at the elbow.

99. For an adult, the normal resting pulse should be between: A) 50 and 60 beats/min. B) 50 and 70 beats/min. C) 60 and 100 beats/min. D) 70 and 110 beats/min.

100. Which of the following statements regarding the secondary assessment is correct? A) The secondary assessment should focus on a certain area or region of the body as determined by the chief complaint. B) The secondary assessment should be performed en route to the hospital, regardless of the severity of the patient's condition. C) A secondary assessment should be performed, even if you must continually manage life threats that were identified in the primary assessment. D) During the secondary assessment, the EMT's primary focus should be on taking the patient's vital signs and obtaining a SAMPLE history.

Page 18

Answer Key
1. 2. 3. 4. 5. 6. 7. 8. 9. 10. 11. 12. 13. 14. 15. 16. 17. 18. 19. 20. 21. 22. 23. 24. 25. 26. 27. 28. 29. 30. 31. 32. 33. 34. 35. 36. 37. 38. 39. 40. 41. 42. 43. 44. D D A B B A B A D A B B B D A B D C C C B D D A D D D D C B C B A A D B A A D A D B B B

Page 19

45. 46. 47. 48. 49. 50. 51. 52. 53. 54. 55. 56. 57. 58. 59. 60. 61. 62. 63. 64. 65. 66. 67. 68. 69. 70. 71. 72. 73. 74. 75. 76. 77. 78. 79. 80. 81. 82. 83. 84. 85. 86. 87. 88. 89. 90.

C C A A D C B B A C C D D D B D B B A D D B C C D B C A D B B B D C C A B B B A A A C C A C

Page 20

91. 92. 93. 94. 95. 96. 97. 98. 99. 100.

D B C C D B A A C A

Page 21

Das könnte Ihnen auch gefallen